Jump to content

Wikipedia:Teahouse: Difference between revisions

From Wikipedia, the free encyclopedia
Content deleted Content added
Replaced content with 'fffffffffffffff'
Tags: Replaced Reverted
Line 1: Line 1:
fffffffffffffff
{{Short description|Community Q&A hub for helping new editors}}
{{skip to top and bottom}}
{{User:MiszaBot/config
|archiveheader = {{Automatic archive navigator}}
|maxarchivesize = 400K
|counter = 1225
|minthreadsleft = 10
|minthreadstoarchive = 2
|algo = old(48h)
|archive = Wikipedia:Teahouse/Questions/Archive %(counter)d
}}
{{clear}}
{{Wikipedia:Teahouse/Header}}

<!-- Questions go here. Please post new questions at the BOTTOM of the page. -->

== Disputed Article ==

Hello. Is it possible to get a review of this disputed article for [[Harry L. Williams]]? I have made edits to my previous edits in an attempt to comply, and those have been further edited. The most recent comment was from OrangeMike who said, "This reads like it was written by Williams' press agent." I reached out to him a few weeks ago to ask what triggered his comment but have not heard back. I am working on behalf of Williams' current employer: Thurgood Marshall College Fund. I've disclosed all related information as a paid subcontractor. I'm not disagreeing, I simply want to get this article in good standing. I can guess what might be causing the issues, but an objective review would really help define this for me if at all possible. Thank you! [[User:Sigridtx|Sigridtx]] ([[User talk:Sigridtx|talk]]) 20:55, 20 May 2024 (UTC)
:Hello, {{u|Sigridtx}} The sentence {{tpq|Under his guidance, TMCF has created innovative partnerships, grown organizational stability, and advanced HBCU bipartisan support}} is promotional, hagiographic and entirely unreferenced. Who says that? You? That is the sort of thing that {{u|OrangeMike}} was probably referring to. [[User:Cullen328|Cullen328]] ([[User talk:Cullen328|talk]]) 21:59, 20 May 2024 (UTC)
::Thank you Cullen328. I actually had that sentence flagged myself and appreciate you confirming that, as well as your response. [[User:Sigridtx|Sigridtx]] ([[User talk:Sigridtx|talk]]) 12:20, 21 May 2024 (UTC)

:Hello, {{u|Sigridtx}}. What Cullen328 says -- though he has only pointed out one particularly egregious sentence. That aside, most of the references are to this or that among the subject's various employers. (Two are to pieces written by Williams himself.) Indeed, a quick look doesn't show me a single ''disinterested'' source. Where are the reliable, disinterested sources? (Do any exist?) -- [[User:Hoary|Hoary]] ([[User talk:Hoary|talk]]) 00:43, 21 May 2024 (UTC)
::Thank you Hoary, I totally understand. A couple of quick things if you don't mind. In navigating through trying to get this compliant, I reviewed articles for similar subjects representing the Black college community (like Williams). In more than one article their employers' websites were cited but the article wasn't "flagged." Also, if I cite an online article Williams authored in context of "Williams writes opinion-editorials on issues such as higher education... (as stated in the article), can I not cite those? Nothing in disagreement here, just mentioning to get this straight. I appreciate your help! [[User:Sigridtx|Sigridtx]] ([[User talk:Sigridtx|talk]]) 12:42, 21 May 2024 (UTC)
:::Hi {{u|Sigridtx}}. Sources <u>not</u> [[Wikipedia:Independent sources|independent]] of the subject can be cited to support <u>un</u>controversial facts, but cannot be used to support the subject's [[Wikipedia:Notability|Notability]]. (A 'controversial' fact, in this context, is one where a reader ''might'' say "Whoa, I'd like to see confirmation of that", or one where the fact ''has'' been challenged.) Non-independent sources should not be used extensively: most of the contents of an article needs to be a summary of what independent [[Wikipedia:Reliable sources|Reliable sources]] say.
:::Be aware of [[WP:Other stuff exists]]. A great many of Wikipedia's existing 6.8 million articles have deficiencies of varying seriousness (and there are far too few editors to bring about a rapid across-the-board improvement). Their faults do not justify editors allowing articles under active consideration to introduce or continue displaying similar faults. Rather than emulating cherrypicked less-than-great articles that happen to be about similar subjects, you would do better to look at a range of [[WP:Good articles]]. Hope this helps. {The poster formerly known as 878.81.230.195} [[Special:Contributions/94.2.67.173|94.2.67.173]] ([[User talk:94.2.67.173|talk]]) 14:02, 21 May 2024 (UTC)
::::Thank you. I'm working on gathering some hopefully "disinterested" sources, however, a good portion of what I've found either have some tie to education (but not exclusively), and news sources focused on the African-American community. I'll work on this more of course. To my question in my previous reply about citing an online article authored by Williams' (in context of the "Williams writes opinion-editorials on issues..." sentence currently in his article), can I cite sources that aren't directly tied to his employer? I have one right now that needs to change. The other two were published in ''Diverse Magazine''. Thanks again! [[User:Sigridtx|Sigridtx]] ([[User talk:Sigridtx|talk]]) 16:47, 22 May 2024 (UTC)
:::::@[[User:Sigridtx|Sigridtx]], on the sourcing question, you might want to review [[Wikipedia:Notability (academics)]]. AIUI (but I could be wrong), being the president of a ''bona fide'' university means an automatic pass under those rules, even if the only thing cited in the article is a press release from the university. You can ask for help from people who understand the very unusual rules for notability of academics on the talk page for that guideline. [[User:WhatamIdoing|WhatamIdoing]] ([[User talk:WhatamIdoing|talk]]) 07:33, 23 May 2024 (UTC)
::::::Thanks so much for that, I'll look into this! [[User:Sigridtx|Sigridtx]] ([[User talk:Sigridtx|talk]]) 18:45, 23 May 2024 (UTC)

== Best way to handle articles with controversial names ==

Hello,

I have been infrequently working on improving documentation of the 1989 mass expulsion event that occurred in Bulgaria and is little-known.

The issue is that the name itself of this event is controversial. The most commonly accepted name is "[[Big Excursion]]", which is a euphemistic name which originally employed officially by the perpetrators (the Bulgarian Communist Party). You will often find the event referred to by this name and this name only. This is a bit like if "Final Solution" was the most commonly use name to describe the Holocaust. Unsurprisingly, the name "Big Excursion" is frequently rejected by academics and victims/their descendants, though I have not seen concrete alternatives proposed in what I have read, just rejection of that name.

In light of that, I have titled the article Big Excursion so that it actually pops up when someone searches for it, but the primary name used in the article and info box is the purely descriptive name of "1989 forced migration". Within the article, I have used "Big Excursion" in quotes and explained that decision in a terminology section.

Would anyone please provide feedback on this choice and link to any relevant Wikipedia policies? Additionally can anyone link to other articles with controversial names for comparison? In some articles, it seems some of the names are outright rejected whereas in others, a descriptive name is use in line with "1989 forced migration" (e.g. [[January 6 United States Capitol attack]]). I intend to make another push at revising the article soon enough, so would greatly appreciate feedback that I could integrate into that. [[User:Pietrus1|Pietrus1]] ([[User talk:Pietrus1|talk]]) 00:41, 21 May 2024 (UTC)
:[[Wikipedia:Article titles]] is the guideline on article-names. The "§Use commonly recognizable names" portion says in general to use "the name that is most commonly used (as determined by its prevalence in a significant majority of independent, reliable, English-language sources)". So if there is only one widely used name, that's the article name. If there is scholarly commentary that the name is inappropriate, that's good article content. But if there's no specific alternative name that's at least somewhat-widely used, we might be stuck with the poor one. [[User:DMacks|DMacks]] ([[User talk:DMacks|talk]]) 03:03, 21 May 2024 (UTC)
::Thanks.
::I will note that in the talk section as well. [[User:Pietrus1|Pietrus1]] ([[User talk:Pietrus1|talk]]) 03:49, 21 May 2024 (UTC)

:[[User:Pietrus1|Pietrus1]], if {{Olive|"Big Excursion" [...] is a euphemistic name which originally employed officially by the perpetrators}}, well, "[[ethnic cleansing]]" is a name that originated as a euphemism employed by the perps. All things being equal, Wikipedia should avoid euphemism, but one reason to tolerate (perhaps within [[scare quotes]]) a euphemistically intended term such as "ethnic cleansing" is that a good understanding of its actual reference can fairly soon outweigh any effect of its euphemistic intention. -- [[User:Hoary|Hoary]] ([[User talk:Hoary|talk]]) 04:30, 21 May 2024 (UTC)

::To give another example: "[[The Troubles]]" is also both a euphemism and the common name for that period of conflict, and it wouldn't really make sense to call that article anything else. -- [[User:DandelionAndBurdock|D'n'B]]-''[[User_talk:DandelionAndBurdock|t]]'' -- 18:31, 21 May 2024 (UTC)
:::Both of these are good examples and make sense. I suppose I am just particularly peeved by this name given that it is referring to crimes against humanity with a propaganda term. We also already had a user modify the page to something not using such a term. [[User:Pietrus1|Pietrus1]] ([[User talk:Pietrus1|talk]]) 21:41, 21 May 2024 (UTC)
::::The most important thing isn't the big page title, but what you write in the article. Your first sentence could include both, perhaps sounding something like "'''The Big Excursion''' was the '''1989 forced migration''' of Bulgarian Muslims..." You could also consider [[Forced migration of Bulgarian Muslims]] as a title. The policy permits multiple options, and you will have to use your best judgment to find the one that works best for the article. [[User:WhatamIdoing|WhatamIdoing]] ([[User talk:WhatamIdoing|talk]]) 07:37, 23 May 2024 (UTC)

== stolen work ==

Walter I, Count of Brienne problem

i created a page for him the page got removed even though in the talks i asked for help about sources and references and for people to help and contribute back on sunday

today i find out that someone else yesterday created a page for him while my page was still in review

i need help to fix this problem and how to keep this from happening again

thanks [[User:Briannemartindale|Briannemartindale]] ([[User talk:Briannemartindale|talk]]) 19:20, 21 May 2024 (UTC)
:{{u|Briannemartindale}}, this is a collaborative project and anyone can work on any topic at any time. There is no such thing as "stolen work" in the sense that you are using that term. Please read [[WP:OWNERSHIP]]. To prevent it from happening again, write well referenced content that establishes notability. [[User:Cullen328|Cullen328]] ([[User talk:Cullen328|talk]]) 19:32, 21 May 2024 (UTC)
::To clarify, [[Walter I, Count of Brienne]] is vastly better than your [[Draft:Walter I of Brienne]]. [[User:Cullen328|Cullen328]] ([[User talk:Cullen328|talk]]) 19:37, 21 May 2024 (UTC)
:In addition to the other replies, since this is a collaborative project, see what you can merge from your draft into the article. ~[[User:Anachronist|Anachronist]] <small>([[User talk:Anachronist|talk]])</small> 19:39, 21 May 2024 (UTC)
::Articles are similar and the mainspace one's first revision does not look like a page creation. {{u|Kansas Bear}} may have opened OP's article to improve it before it got draftified and published the improved version after the draftification. [[WP:HISTMERGE]] would be best if so. When you've only written one article so far, it hurts more when first revision goes to someone else than after you've created dozen or a hundred. So, I wouldn't be too hard on OP. Best,<span id="Usedtobecool:1716321735125:WikipediaFTTCLNTeahouse" class="FTTCmt"> —&nbsp;'''[[User:Usedtobecool|Usedtobecool]]'''&nbsp;[[User talk:Usedtobecool|☎️]] 20:02, 21 May 2024 (UTC)</span>
:::Actually, I was unaware that Briannemartindale had a draft of Walter I of Brienne. I took the French version[https://fr.wikipedia.org/wiki/Gautier_Ier_de_Brienne], translated(what I could), then started adding references. I did the same for [[Guy of Bar-sur-Seine]]. Judging from this [[County_of_Brienne#Counts_of_Brienne|list]], there are plenty of Briennes. --[[User:Kansas Bear|Kansas Bear]] ([[User talk:Kansas Bear|talk]]) 20:37, 21 May 2024 (UTC)
::::It's just an unlikely coincidence, then. [[User:Briannemartindale|Briannemartindale]], I've been involved in these sorts of incidents once or twice myself. It happens. Stay around long enough, and you may end up doing this to another editor too. I would recommend you just redirect the draft to the existing article and move on. Best,<span id="Usedtobecool:1716340087042:WikipediaFTTCLNTeahouse" class="FTTCmt"> —&nbsp;'''[[User:Usedtobecool|Usedtobecool]]'''&nbsp;[[User talk:Usedtobecool|☎️]] 01:08, 22 May 2024 (UTC)</span>
::::you didn't know
:::::im not mad i just wanted to know how to keep this from happening again that's all [[User:Briannemartindale|Briannemartindale]] ([[User talk:Briannemartindale|talk]]) 15:02, 22 May 2024 (UTC)
:::::: Short answer: you can't prevent someone else from creating an article while your article is in draft status. Just because you were first (to create a draft) does not give you [[WP:OWNERSHIP|ownership]] over the topic; Wikipedia topics/articles ''have no owners''.

:::::: If you have content that is missing from the article in mainspace (the published article), add it. If you have (good) sources/citations that are missing, add those. If you think your wording about a particular point of the topic is better than the wording currently in the published version, change the wording. Just remember that you're ''offering'' your changes to all other interested Wikipedia editors; if they are good changes, they are likely to stay, but if someone doesn't agree that the changes are better, you'll probably need to discuss that (on the article's Talk page), and see if a majority of (interested) editors agree with you. Or to see if you can reach a good compromise. -- <span style="font-family:Brush Script MT; font-size:15px;">[[User:John Broughton|John Broughton]] </span> [[User talk:John Broughton |(♫♫)]] 23:56, 22 May 2024 (UTC)

== Question ==

Hello Teahouse, weird question but..is there an age limit to edit on Wikipedia? [[User:GoodHue291|GoodHue291]] ([[User talk:GoodHue291|talk]]) 19:33, 21 May 2024 (UTC)

:No, anyone can edit, although see also [[Wikipedia:Competence is required]]. I have even encountered an administrator who was a teenager. It can happen if an editor shows maturity and responsible behavior. ~[[User:Anachronist|Anachronist]] <small>([[User talk:Anachronist|talk]])</small> 19:37, 21 May 2024 (UTC)
::{{u|GoodHue291}}, I suggest that you read [[Wikipedia:Guidance for younger editors]]. [[User:Cullen328|Cullen328]] ([[User talk:Cullen328|talk]]) 19:41, 21 May 2024 (UTC)
:::This is what I needed. Thanks Cullen! [[User:GoodHue291|GoodHue291]] ([[User talk:GoodHue291|talk]]) 19:42, 21 May 2024 (UTC)
:::Well actually before I forget I had another question (to prevent myself from creating another thread), is there a webpage here where you can do your test edits? I'm not talking about a sandbox it's another thing on here but I forget what it is. [[User:GoodHue291|GoodHue291]] ([[User talk:GoodHue291|talk]]) 21:36, 21 May 2024 (UTC)
::::Hi, GoodHue291. I'm not sure what you mean: there are actually two different things called sandbox: the Wikipedia sandbox ([[WP:SANDBOX]]) which anybody can use, and you can practise editing, but it gets cleared regularly (every day I think, but I'm not sure).
::::There's also your personal sandbox [[User:GoodHue291/sandbox]] , which I see you have found. You can edit and put anything in as long as it is in accordance with [[WP:UPYES]], and as long as it doesn't break those conditions, nobody will delete it. It is typically used for developing articles, but you can use it for other purposes, including practising editing.
::::Or were you thinking of [[WP:The Wikipedia Adventure|The Wikipedia Adventure]]? [[User:ColinFine|ColinFine]] ([[User talk:ColinFine|talk]]) 21:57, 21 May 2024 (UTC)
:::::I believe I was looking for [[Wikipedia:Sandbox|this]], thanks man. [[User:GoodHue291|GoodHue291]] ([[User talk:GoodHue291|talk]]) 22:23, 21 May 2024 (UTC)
::But [[User:Anachronist|Anachronist]], do we have centenarian editors? -- [[User:Hoary|Hoary]] ([[User talk:Hoary|talk]]) 22:43, 21 May 2024 (UTC)
:::I'm sure we do. I started editing about 18 years ago and encountered an editor in his 80s, so if he's still around, he'd be centenarian. It's a small population. I'm only a, um, hexenarian? myself, probably about {{U|Cullen328}}'s age based on his userpage picture. ~[[User:Anachronist|Anachronist]] <small>([[User talk:Anachronist|talk]])</small> 14:12, 22 May 2024 (UTC)
::::(I believe the most common word for people in their 60s is ''sexagenarian''.) [[User:WhatamIdoing|WhatamIdoing]] ([[User talk:WhatamIdoing|talk]]) 07:43, 23 May 2024 (UTC)
:Incidentally, [[User:GoodHue291|GoodHue291]], it's a good idea to give a new message thread a title that, unlike "Question", is informative and doesn't duplicate that of an earlier thread on the same page. -- [[User:Hoary|Hoary]] ([[User talk:Hoary|talk]]) 22:45, 21 May 2024 (UTC)
::You can see the difference if you click on the threads. The thread or a header, should be the main topic of your question, followed by a message about it. I wasn't sure what to put as a header, so I went with question, because it's the main basis of my writing in the beggining. [[User:GoodHue291|GoodHue291]] ([[User talk:GoodHue291|talk]]) 18:43, 22 May 2024 (UTC)

== reliable resource ==

how to provide a reliable resource? [[User:Ellpasha|Ellpasha]] ([[User talk:Ellpasha|talk]]) 21:31, 21 May 2024 (UTC)

:Hello @[[User:Ellpasha|Ellpasha]]! A '''[[WP:RS|reliable source]]''' is a source that is trustworthy, credible, and can be verified. It might be a book, a website, a film, or something else, but [[WP:SELFPUB|self-published sources]] like blogs and social media are generally not reliable, as are websites, newspapers, and publishers with a poor reputation for [[fact-checking]]. [[User:Cremastra|Cremastra]] ([[User talk:Cremastra|talk]]) 21:57, 21 May 2024 (UTC)
:Hello Ellpasha. Please look at [[WP:reliable sources]] for information about that. [[User:ColinFine|ColinFine]] ([[User talk:ColinFine|talk]]) 21:58, 21 May 2024 (UTC)
::A related question, @[[User:ColinFine|ColinFine]] ...
::In a recent Teahouse discussion, a resource entitled [[Wikipedia: Biographies of living persons]] was suggested to an editor by another (presumably senior) editor. I took a look at that resource and there found a stunning piece of information: that it's okay to cite material from a personal website of a living person writing about himself.
::It this is so, it beautifully solves a dilemma I've been facing about how to cite a few facts about a living person that no third party seems to have published. But it's been so hammered into us that only third-party material can be used that I keep hoping my eyes weren't playing tricks on me!.
::I'd love to see verification about this from you as a trusted senior editor. [[User:Augnablik|Augnablik]] ([[User talk:Augnablik|talk]]) 01:35, 23 May 2024 (UTC)
:::Per [[WP:ABOUTSELF]], it's okay to cite self-published sources if the subject is about themselves, with some reasonable caveats. [[User:Cremastra|Cremastra]] ([[User talk:Cremastra|talk]]) 01:41, 23 May 2024 (UTC)
::::Thank you , @[[User:Cremastra|Cremastra]] … you beat Colin to verify what I was asking about. But now a follow-up question: why is such a gem as this permission so buried in Wikipedia’s editorial caverns? I might never have come across it had I not followed up in curiosity about a resource suggested to ''another'' editor. [[User:Augnablik|Augnablik]] ([[User talk:Augnablik|talk]]) 01:58, 23 May 2024 (UTC)

== Geçmişten günümüze beyoğlu belediye başkanları ==

1860 tan 2024 e kadar Beyoğlu belediye başkan listesi [[Special:Contributions/78.183.149.105|78.183.149.105]] ([[User talk:78.183.149.105|talk]]) 14:25, 22 May 2024 (UTC)

:According to Google Translate, this request is about a list of mayors of [[Beyoğlu]]. What specifically is your request? [[User:QuicoleJR|QuicoleJR]] ([[User talk:QuicoleJR|talk]]) 14:28, 22 May 2024 (UTC)
*Şu başkanlar listesi ikisi de İngilizcenin Vikipediya veya Türkçenin Vikipediya yok. <sub>signed, </sub>[[User:Rosguill|'''''Rosguill''''']] <sup>[[User talk:Rosguill|''talk'']]</sup> 15:41, 23 May 2024 (UTC)

== How to get a mentor? ==

Hello all. I see that some new editors are assigned a mentor to be there as a guide, support, etc. Such a cool idea. How are folks assigned to a mentor, do you have to sign up? Do you just reach out to an editor and ask? Thanks :) [[User:Taevchoi|Taevchoi]] ([[User talk:Taevchoi|talk]]) 16:15, 22 May 2024 (UTC)

:@[[User:Taevchoi|Taevchoi]] We only have enough mentors for about 50% of new accounts to get them at present. However, if you were in the half that didn't, then you can activate the Newcomer Homepage at [[Special:preferences]] (at the bottom there is a check-box). Once you have saved that change in your preferences, the homepage tab will be visible when you navigate to your userpage and that tab has the name of your assigned mentor. [[User:Michael D. Turnbull|Mike Turnbull]] ([[User talk:Michael D. Turnbull|talk]]) 17:31, 22 May 2024 (UTC)
::Thank you! [[User:Taevchoi|Taevchoi]] ([[User talk:Taevchoi|talk]]) 18:09, 22 May 2024 (UTC)
::@[[User:Michael D. Turnbull|Michael D. Turnbull]], are you saying a new editor can "force" themselves a mentor this way? [[User:Gråbergs Gråa Sång|Gråbergs Gråa Sång]] ([[User talk:Gråbergs Gråa Sång|talk]]) 18:46, 22 May 2024 (UTC)
:::@[[User:Gråbergs Gråa Sång|Gråbergs Gråa Sång]] Yes. I am a mentor and wanted to check out how the newcomer homepage tab worked and what it looked like: in activating it on my account I was assigned a mentor (whom I have never contacted). [[User:Michael D. Turnbull|Mike Turnbull]] ([[User talk:Michael D. Turnbull|talk]]) 10:52, 23 May 2024 (UTC)
::::Come to think of, I did that too. [[User:Gråbergs Gråa Sång|Gråbergs Gråa Sång]] ([[User talk:Gråbergs Gråa Sång|talk]]) 10:58, 23 May 2024 (UTC)
::@[[User:Michael D. Turnbull|Michael D. Turnbull]], I'm in the 50% of editors who got a mentor. But now, two years after the Wiki start gate opened for me, I feel that so many senior editors have been like mentors in the Teahouse and other areas where we can ask questions that I'd be willing to "free up" my assigned mentor. Perhaps other editors would, too. [[User:Augnablik|Augnablik]] ([[User talk:Augnablik|talk]]) 01:43, 23 May 2024 (UTC)
:::@[[User:Augnablik|Augnablik]] Yes, I made exactly that point to the Growth Team at [[WP:Growth Team features/Mentor list#Suggestion to "retire" mentees]] on 15 April. That team has taken up the suggestion but it is not yet implemented (see that thread). Incidentally, I find that most of the newcomers who are assigned to me as a mentor never make contact and of those who do, most do so only once. [[User:Michael D. Turnbull|Mike Turnbull]] ([[User talk:Michael D. Turnbull|talk]]) 10:58, 23 May 2024 (UTC)
::::It can also happen that assigned mentors don’t really connect with their mentees. [[User:Augnablik|Augnablik]] ([[User talk:Augnablik|talk]]) 12:05, 23 May 2024 (UTC)
:::::I don't actively check who is being assigned to me. I await their contact, which happens on my Talk Page. I currently have 625 mentees, with new ones being added at 2 to 4 per day. If I added a welcome message to all of them I would have little time to do anything else! [[User:Michael D. Turnbull|Mike Turnbull]] ([[User talk:Michael D. Turnbull|talk]]) 13:51, 23 May 2024 (UTC)
::::::You have 625 mentees? Dear God!
::::::Is that the typical ratio per senior editor? [[User:Augnablik|Augnablik]] ([[User talk:Augnablik|talk]]) 14:29, 23 May 2024 (UTC)
:::::::Yes, my mentor dashboard is set so I get the "average" number. This perhaps explains why only 50% of new users on enWiki get the newcomer homepage. Note that, as I've already said, few of my assigned mentees ever contact me: about 30 have done so this year (see my Talk Page: they are the ones with timestamps in the section title, which is how this newcomer feature works). [[User:Michael D. Turnbull|Mike Turnbull]] ([[User talk:Michael D. Turnbull|talk]]) 14:38, 23 May 2024 (UTC)

::::The idea of assigned mentors is a nice “warm fuzzy,” and appreciated as we start out on our Wiki editing journey making our way through the fog.
::::But if (1) there aren’t enough mentors to go around; and (2) mentor-mentee interaction isn’t as strong as anticipated when the program was initiated; and (3) mentees find good support from non-assigned senior editors simply through discussion like here in the Teahouse, perhaps assigned mentors aren’t really needed.
::::But ''something'' is. For awhile, at least. What about occasional Internet forums via threaded message boards. These could be available for all new editors to take part in, within some sort of time frame like 6 months or a year after they come to Wikipedia.
::::Since newbies would interact with several senior editors rather than just one — and in addition, with fellow newbies — this could actually expand their sense of connection with Wikipedia beyond what they have in the current assigned mentor arrangement. [[User:Augnablik|Augnablik]] ([[User talk:Augnablik|talk]]) 13:13, 23 May 2024 (UTC)
:::Or the other way round :). [[User:Lectonar|Lectonar]] ([[User talk:Lectonar|talk]]) 12:33, 23 May 2024 (UTC)
::Seems that some accounts don't have mentorship, even with the homepage. (Homepage seems to be rolled out to all accounts.) All accounts technically do have an assigned mentor, but the panel is not visible to the other half of the new accounts. However, based on my past testing, "claiming" a mentee (from a mentor's dashboard) makes the panel visible, as I did on my own account. ~~[[User:2NumForIce|<span style="background:#007eb3;color:#ffffff;">2NumForIce</span>]] ([[User talk:2NumForIce|<sup style="color:#007eb3">speak</sup>]]<span style="color:55a7db">&#124;</span>[[Special:Contributions/2NumForIce|<sub>edits</sub>]]) 15:18, 25 May 2024 (UTC)

== spelling and grammar help ==

i need help

i have suffered from many strokes several years ago and have recovered as best as i can

i am now trying to help with editing and creating new articles on wiki to help everyone else in the world but some of articles are getting removed due to spelling and grammar issues even though on the talk page of the new article i have asked for help with making it look better

spelling and grammar have not been my strong suits especially after my strokes

what can i do because i am getting tired of my pages getting removed

ps i think its only one person removing my pages

thanks [[User:Briannemartindale|Briannemartindale]] ([[User talk:Briannemartindale|talk]]) 17:43, 22 May 2024 (UTC)
:Hello, {{u|Briannemartindale}}. I have looked at two of your articles that were turned into drafts, and the problems were not spelling and grammar. The problems were that the articles in question failed to make a convincing case that the people were [[WP:NPERSON|notable]] in Wikipedia's definition. I suggest that you develop drafts carefully, taking time to develop them until notability is well-referenced and indisputable, and the articles cleaned up. Then, submit your drafts through the [[WP:AFC|Articles for Creation]] process. That should result in less frustration. [[User:Cullen328|Cullen328]] ([[User talk:Cullen328|talk]]) 18:24, 22 May 2024 (UTC)
::i was going by what the notes said to me about my drafts [[User:Briannemartindale|Briannemartindale]] ([[User talk:Briannemartindale|talk]]) 18:33, 22 May 2024 (UTC)
:::I only see one note on your user page that mentions language or grammar. However, I did learn that you are trying to translate articles from French to English even though you state {{tpq|i dont speak much french}}. This is a ''really bad idea''. You should not be translating any articles unless you are highly proficient in both languages. Please stop. [[User:Cullen328|Cullen328]] ([[User talk:Cullen328|talk]]) 18:47, 22 May 2024 (UTC)
::::i am only translating basic information to get others a starting point on a person of royalty or nobility
::::if you want to help improve my articles you can and the help would be much appreciated and would be more helpful and productive for everyone than to remove the articles
::::thanks for your understanding [[User:Briannemartindale|Briannemartindale]] ([[User talk:Briannemartindale|talk]]) 19:00, 22 May 2024 (UTC)
:::::Hi @[[User:Briannemartindale|Briannemartindale]]. As far as I can tell none of your articles have been ''removed'', they were instead moved to draft as they were not ready for the project.
:::::I have just declined your draft [[Draft:Sir_Richard_Wentworth]] as I see no evidence this person is notable by our standards. <span style="background-color: RoyalBlue; border-radius: 1em; padding: 3px 3px 3px 3px;">'''[[User:Qcne|<span style="color: GhostWhite">Qcne</span>]]''' <small>[[User talk:Qcne|<span style="color: GhostWhite">(talk)</span>]]</small></span> 19:22, 22 May 2024 (UTC)
::::::it would be much better for everyone to improve on my articles vs declining the drafts for i only work on royalty and nobility and their families
::::::i believe it would be more productive for everyone to improve on the articles vs putting them back to drafts
::::::thanks [[User:Briannemartindale|Briannemartindale]] ([[User talk:Briannemartindale|talk]]) 19:39, 22 May 2024 (UTC)
:::::::@[[User:Briannemartindale|Briannemartindale]] but Wikipedia has notability requirements, and if you are creating articles about people who are not notable, then they likely cannot be improved. No amount of editing can confer notability; only significant coverage in multiple reliable secondary sources can. So far in your two articles, neither person has any hint of notability. <span style="background-color: RoyalBlue; border-radius: 1em; padding: 3px 3px 3px 3px;">'''[[User:Qcne|<span style="color: GhostWhite">Qcne</span>]]''' <small>[[User talk:Qcne|<span style="color: GhostWhite">(talk)</span>]]</small></span> 19:41, 22 May 2024 (UTC)
::::::::father of a baron and uncle to a queen of england
::::::::here is the notability of the person [[User:Briannemartindale|Briannemartindale]] ([[User talk:Briannemartindale|talk]]) 19:59, 22 May 2024 (UTC)
:::::::::I'm afraid that being uncle to a queen is NOT part of the [[WP:GNG|notability]] criteria. [[User:Theroadislong|Theroadislong]] ([[User talk:Theroadislong|talk]]) 20:01, 22 May 2024 (UTC)
::::::::::that is your opinion and each person's opinion is going to be different and i respect that and i will try and get the article moved forward anyway thanks [[User:Briannemartindale|Briannemartindale]] ([[User talk:Briannemartindale|talk]]) 20:12, 22 May 2024 (UTC)
:::::::::::@[[User:Briannemartindale|Briannemartindale]] I am afraid that is not opinion, that is Wikipedia policy. Please read [[WP:NPEOPLE]]. <span style="background-color: RoyalBlue; border-radius: 1em; padding: 3px 3px 3px 3px;">'''[[User:Qcne|<span style="color: GhostWhite">Qcne</span>]]''' <small>[[User talk:Qcne|<span style="color: GhostWhite">(talk)</span>]]</small></span> 20:18, 22 May 2024 (UTC)
:::::::::::Briannemartindale, you're wrong. Your opinion or my opinion about notability don't matter a bit. See also [[WP:NOTINHERITED|notability is not inherited]]. Wikipedia has [[WP:GNG|objective standards]] to determine notability and these standards are not based on anyone's personal opinion, but by community consensus.
:::::::::::I recommend you '''STOP''' creating articles about non-notable subjects expecting others to improve them, because that is not possible to do if a subject isn't notable. You need to demonstrate '''first''' that the subject is notable before '''anyone''' would consider letting the article exist in main space, let alone improving them. ~[[User:Anachronist|Anachronist]] <small>([[User talk:Anachronist|talk]])</small> 21:31, 22 May 2024 (UTC)
{{od|12}} Hi {{u|Briannemartindale}}. Are you aware of [[:Wikipedia:WikiProject Royalty and Nobility]]? The members of that particular [[:WP:WPPJ|WikiProject]] seem to be interested in things that you're interested in; so, they might be good people to ask for assistance when it comes to types of articles you're trying to create. I've posted on that [[:Wikipedia talk:WikiProject Royalty and Nobility#User:Briannemartindale and articles about royals/nobles|WikiProject's talk page]] to see if any of its members might be willing to sort of mentor you and help you with the issues you've been having. In the meantime, I suggest that perhaps you refrain in creating any more new articles directly in the mainspace, but instead work on improving those which have been draftified. While it's true that Wikipedia wants us to be [[:WP:BOLD]] when we edit (including when it comes to creating new articles), creating too many one or two sentence ones with inadequate sourcing or unclear claims of Wikipedia notability isn't a good idea; moreover, subsequently posting on the article's talk asking others to improve things for you is most likely not going to get many favorable responses. It's not a good sign when the articles you're creating are constantly being draftified because that pretty much always has to do with [[:WP:NOTABILITY]] and [[:WP:OVERCOME|not spelling and grammar]]. Maybe also take a look at [[:Help:Your first article]] and [[:Help:Referencing for beginners]] for some tips on how to write articles and how to add citations to articles. Anyway, continue to work on your drafts and submit them for review to [[:Wikipedia: Articles for creation]] when you think they're ready. Perhaps once you've established a better track record for creating proper articles, you might find the process to be a little less bumpy. -- [[User:Marchjuly|Marchjuly]] ([[User talk:Marchjuly|talk]]) 21:44, 22 May 2024 (UTC)
== AGE ==

Question, Can children below 13 can edit, make articles? [[User:Nedia020415|NeD1a &#91;&#91;User:Nedia020415&#124;Nedia020415&#93;]] ([[User talk:Nedia020415|talk]]) 19:34, 22 May 2024 (UTC)

:{{ping|Nedia020415}} Absolutely! [[Wikipedia:Guidance for younger editors]] may be a useful page to read as well. [[User:Tollens|Tollens]] ([[User talk:Tollens|talk]]) 19:37, 22 May 2024 (UTC)
::Thank you @[[User:Tollens|Tollens]]! [[User:Nedia020415|NeD1a &#91;&#91;User:Nedia020415&#124;Nedia020415&#93;]] ([[User talk:Nedia020415|talk]]) 15:52, 23 May 2024 (UTC)

== Inquery about a certain article ==

Hello Teahouse,

I came here to ask about [[Marwan Abdullah Abdulwahab Noman|this article in particular]] about its [[Wikipedia:Notable|notability]] and it's [[Wikipedia:Verifiability|singled sourced content]]. I'm not sure the right course of action for this to whether [[Wikipedia:Ignore all rules|leave it alone]], nominate it [[Wikipedia:Articles for deletion|for deletion]], or look for more sources on the article. I'm stuck here. [[User:GoodHue291|GoodHue291]] ([[User talk:GoodHue291|talk]]) 20:35, 22 May 2024 (UTC)

:{{ping|GoodHue291}} If you think that it may not be notable, you should look for more sources. If you find enough sources, it is notable, and you should add those sources to the article. If you can't find enough sources, nominate the page for deletion. Make sure you don't just use Google and you also check Google News, Google Books, and Google Scholar. If you do not have time to check, you can tag the page with [[Template:Notability]], which will add it to the category of articles that might not be notable so that someone else can eventually come and check the article. I hope this helps. [[User:QuicoleJR|QuicoleJR]] ([[User talk:QuicoleJR|talk]]) 21:31, 22 May 2024 (UTC)
:Hello, @GoodHue291. I have tagged the article as needing more sources and possibly not meeting the criteria for [[WP:notability|notability]]; I have also tagged the sole reference with {{tl|failed verification}}, since it doesn't actually mention Noman or the position he is said to have resigned from.
:The ideal course of action is to look for sources, and either add them or nominate the article for deletion ([[WP:AFD]]) if they can't be found. But this is not an insignificant task, so people often don't do it, but just do a "drive-by tagging", as I have done - which at least alerts a reader to the fact that there are doubts about the article. [[User:ColinFine|ColinFine]] ([[User talk:ColinFine|talk]]) 21:41, 22 May 2024 (UTC)
::Actually, the source does mention Marwan Abdullah Abdulwahab Noman, in the last sentence under "Exchange of Diplomatic Representation". It doesn't mention his resigning, though. [[User:Deor|Deor]] ([[User talk:Deor|talk]]) 23:17, 22 May 2024 (UTC)
:::Oops, I see that the current source was added after you posted the message above. Nonetheless, it verifies only that he was appointed as ambassador in 2007. [[User:Deor|Deor]] ([[User talk:Deor|talk]]) 23:24, 22 May 2024 (UTC)
::::I've changed the wording accordingly. -- <span style="font-family:Brush Script MT; font-size:15px;">[[User:John Broughton|John Broughton]] </span> [[User talk:John Broughton |(♫♫)]] 00:03, 23 May 2024 (UTC)

== Changing pronouns ==

Hey! I've been editing Wikipedia for four or so years now, but I've been publicly out as a trans guy for three and just realized my pronouns have been wrong in my profile for a long time. I'm having trouble switching them over, though—I've been trying to change them in Preferences but they keep reverting back to feminine pronouns. Anyone know how to fix this? Thanks! [[User:TariffedSparrow|TariffedSparrow]] ([[User talk:TariffedSparrow|talk]]) 20:57, 22 May 2024 (UTC)

:{{ping|TariffedSparrow}} Just to be sure, are you pressing Save after changing the pronouns? [[User:QuicoleJR|QuicoleJR]] ([[User talk:QuicoleJR|talk]]) 21:32, 22 May 2024 (UTC)
:Try setting it to "unspecified" first and see if that sticks. If it does, set it again how you want. Oh, and as QuicoleJR said, be sure to click the Save button at the bottom. You can't simply simply change them, you must also save them. ~[[User:Anachronist|Anachronist]] <small>([[User talk:Anachronist|talk]])</small> 21:33, 22 May 2024 (UTC)
::@[[User:QuicoleJR|QuicoleJR]]@[[User:Anachronist|Anachronist]] I've been hitting save but it's still not working :/ Unspecified doesn't work either [[User:TariffedSparrow|TariffedSparrow]] ([[User talk:TariffedSparrow|talk]]) 21:42, 22 May 2024 (UTC)
:::Huh, that's weird. I just tested it and it works fine for me. I'm not sure what the issue is. [[User:QuicoleJR|QuicoleJR]] ([[User talk:QuicoleJR|talk]]) 21:45, 22 May 2024 (UTC)
:::Try from a different browser or device, try flushing your browser cache, try it from an incognito window. You may have a cookie that's stuck or a caching issue on your end. ~[[User:Anachronist|Anachronist]] <small>([[User talk:Anachronist|talk]])</small> 21:49, 22 May 2024 (UTC)
:::Do other settings not save either, or just this one? &ndash; [[Special:Contributions/2804:F14:80E4:8401:960:2088:A68A:DF55|2804:F14:80E4:8401:960:2088:A68A:DF55]] ([[User talk:2804:F14:80E4:8401:960:2088:A68A:DF55|talk]]) 22:31, 22 May 2024 (UTC)
:{{ping|TariffedSparrow}} This is how it ''should'' work:
# [[Special:Preferences]] initially has a grey "Save" button at the bottom.
# Click the circle next to "Use masculine terms when possible" and the circle turns blue.
# The Save button is now blue.
# Click the Save button and it turns grey.
# Your new preference is now saved and "Use masculine terms when possible" is already selected when you visit [[Special:Preferences]] in the future.
:If it still fails then which part goes wrong? [[User:PrimeHunter|PrimeHunter]] ([[User talk:PrimeHunter|talk]]) 23:01, 22 May 2024 (UTC)
::I'm not sure whether they would prevent overwriting local preferences, but you could look at your [[:meta:Special:GlobalPreferences|Global Preferences]] and check whether you have your pronouns set there. If not, try updating your global preferences and see if it (a) sticks and (b) affects this wiki. [[User:Adam Black GB|<span style="color:red">Adam</span> <span style="color:blue">Black</span>]] <sup>[[User talk:Adam Black GB|<span style="color:green">t</span>]] &bull; [[Special:Contributions/Adam_Black_GB|<span style="color:orange">c</span>]]</sup> 00:24, 23 May 2024 (UTC)

== Does this count as a conflict of interest? ==

I was looking through recent edits and found an IP user that has only made edits about universities in their country. When I did a whois lookup on the IP address it showed as belonging to a government agency that helps universities communicate and collaborate on research. I know that if the editor worked for the universities it would be a COI but I'm unsure about a government agency that was made to work with universities. Does this count as a conflict of interest and if they're doing it as part of their job does it count as paid editing? [[User:RomeshKubajali|RomeshKubajali]] ([[User talk:RomeshKubajali|talk]]) 23:04, 22 May 2024 (UTC)
: In my opinion, that depends on the type of edit. If what is being added are things like university size, university history, or major degree programs, that wouldn't seem to me to be [[WP:COI|a conflict of interest]]. If, on the other hand, what is being added is text like "The university has major research collaborations with X, on the topic of Y, and with Z, on the topic of A, and is looking to collaborate with other universities regarding B, C, and D", then yes, that looks like a COI violation. -- <span style="font-family:Brush Script MT; font-size:15px;">[[User:John Broughton|John Broughton]] </span> [[User talk:John Broughton |(♫♫)]] 00:09, 23 May 2024 (UTC)
::That makes sense. Thanks for the help. [[User:RomeshKubajali|RomeshKubajali]] ([[User talk:RomeshKubajali|talk]]) 03:11, 23 May 2024 (UTC)

== The use of euphemisms and weasel words? ==

I have a question about Wikipedia style policies on the use of euphemism and weasel words, in particular as used in article titles.  Eg. is it inappropriate to use the "unrest" as a euphemism for "riots". Do such style policies exist?  Can someone point me to them?

Is Wikipedia:Teahouse to ask this sort of question? [[User:RealLRLee|RealLRLee]] ([[User talk:RealLRLee|talk]]) 23:13, 22 May 2024 (UTC)

:See [[MOS:WEASEL]]. As for whether 'unrest' rather than 'riot' is appropriate in a title, it would depend on the context, and on how appropriate sources described the event in question. [[User:AndyTheGrump|AndyTheGrump]] ([[User talk:AndyTheGrump|talk]]) 23:18, 22 May 2024 (UTC)
::Looking at WP:EUPHEMISM, if the event meets the definition of riot then "riot" is preferred over "unrest". Do I have that right? [[User:RealLRLee|RealLRLee]] ([[User talk:RealLRLee|talk]]) 20:57, 23 May 2024 (UTC)
:::No. The common name is generally preferred over others. Also the definition of something is often not so important to us directly anyway. We generally go by what reliable secondary sources say rather than editor interpretations of whether something meets some definition. Reliable secondary sources will normally rely on definitions but it's not something we're independently deciding. [[User:Nil Einne|Nil Einne]] ([[User talk:Nil Einne|talk]]) 07:16, 25 May 2024 (UTC)

== Draft status for SNL character article ==

I recently created an article that was intended to be a stand-alone page for [[List of recurring Saturday Night Live characters and sketches#1995-1996]]. Because I didn't add citations at first, the article was moved to the draft space. I have since added references and submitted the article for review, but did not get any feedback except for the comment at the top (see [[Draft:Recurring Saturday Night Live characters and sketches introduced 1995–96]]). Not sure what this means, but would welcome any advice on how to improve the article so as not to be rejected. I think the amount of references I've included are sufficient. [[User:Spectrallights|Spectrallights]] ([[User talk:Spectrallights|talk]]) 23:52, 22 May 2024 (UTC)

== How can I join a WikiProject ==

I am very excited as a new editor on Wikipedia and it has given a me new sense of purpose and a unique experience. I want to share my knowledge and information with others as a way of promoting learning. So as a matter of fact, there are numerous articles that are inadequate and lacks important information, which is absolutely unacceptable, that’s why I joined to make sure that none of them be left to obscurity. It would be helpful if I can find others to share my concerns and opinions regarding this issue, I am currently looking for a project to join and seems to have a slight problem. Can anyone help me out with this error? [[User:Davecorbray|Davecorbray]] ([[User talk:Davecorbray|talk]]) 02:30, 23 May 2024 (UTC)

:Hello, @[[User:Davecorbray|Davecorbray]] and welcome to the Teahouse. You can join any WikiProject, no limit to how many, by putting your name in the participants list of the WikiProject. If you also want, you can display that you are in the WikiProject by using [[WP:UBX|userboxes]]! Thank you, <span style="font-family:Arial;background-color:#fff;border:2px dashed#69c73e">[[User:Cowboygilbert|<span style="color:#3f6b39">'''Cowboygilbert'''</span>]] - [[User talk:Cowboygilbert|<span style="color:#d12667"> (talk) ♥</span>]]</span> 02:58, 23 May 2024 (UTC)
::@[[User:Choess|Choess]], what's the best page for finding editors interested in UK parliamentary history? [[Wikipedia:WikiProject Peerage and Baronetage]], [[Wikipedia:WikiProject Biography/Politics and government]], [[Wikipedia:WikiProject Politics of the United Kingdom]], something else?
::@[[User:Davecorbray|Davecorbray]], getting started with a smaller group ("WikiProject" is our jargon for a voluntary group of editors) can be a little daunting some times. If there's not much conversation for you to join in, then you might need to start one yourself, to find out if anyone's [[Help:Watchlist|watching]] that page. You can post a note on the group's talk page with any questions you have, or just to say hello and tell people what's interesting you right now. It may take a bit to get a response, but mostly people are friendly. [[User:WhatamIdoing|WhatamIdoing]] ([[User talk:WhatamIdoing|talk]]) 07:28, 23 May 2024 (UTC)
:::{{Reply to|Davecorbray}} I would probably recommend [[Wikipedia talk:WikiProject Politics of the United Kingdom]] as a starting point, although more of the conversations there are on contemporary politics. A lot of the articles you're looking at probably started out with the text of a short biography (say, from the old Dictionary of National Biography) rather haphazardly expanded as each editor dropped in a few facts of interest with a citation, so they tend to look rather unbalanced. I appreciate the wealth of academic sources you're introducing here. [[User:Choess|Choess]] ([[User talk:Choess|talk]]) 05:15, 24 May 2024 (UTC)
::::Thank you I appreciate your support. [[User:Davecorbray|Davecorbray]] ([[User talk:Davecorbray|talk]]) 10:16, 24 May 2024 (UTC)

== articles getting moved to drafts and criticism ==

hi everyone

i keep getting my articles moved to drafts even with sources/ references

my other issue is that it is the same few people are doing this while everyone else is being very helpful and nice to me and is helping to edit and contribute to my articles

i have picked up criticism from the same few people and they are removing my articles to drafts repeatedly but everyone else is being nice to me and is helping me out

i would like to get this behavior stopped but i don't know how

i need help not criticism

thanks everyone [[User:Briannemartindale|Briannemartindale]] ([[User talk:Briannemartindale|talk]]) 03:33, 23 May 2024 (UTC)
:@[[User:Briannemartindale|Briannemartindale]]: {{welcometea}} I see a fair share of seasoned editors have done you a kindness and drafitied content you started as a way for you to improve on them outside of the main articlespace. I strongly suggest you take a break from creating new articles and work on existing articles, as well as take some glances at [[WP:FA|featured]] and [[WP:GA|good articles]] to see what passes for acceptable content. —[[User:Tenryuu|<span style="color:#556B2F">Tenryuu&nbsp;🐲</span>]]&nbsp;(&nbsp;[[User talk:Tenryuu|💬]]&nbsp;•&nbsp;[[Special:Contributions/Tenryuu|📝]]&nbsp;) 04:15, 23 May 2024 (UTC)
::i need help from others and not be drafitied as you say
::i need help editing the articles and yet the people drafting and criticsing my articles are not helping me edit anything
::i don't know how to get this all to stop without me leaving completely
::thanks [[User:Briannemartindale|Briannemartindale]] ([[User talk:Briannemartindale|talk]]) 04:28, 23 May 2024 (UTC)
:::No one is obligated to help you edit drafts that contain (at the moment) subpar material. Creating articles is one of the hardest things an editor can do on Wikipedia, and the drafting process is a proposed optional pathway where experienced editors can offer constructive criticism. You get this to stop by taking the time to figure out what makes an article acceptable on Wikipedia, and gradually apply those concepts to your drafts, such as [[WP:V|verifiability]], one of the core concepts of this online encyclopedia. —[[User:Tenryuu|<span style="color:#556B2F">Tenryuu&nbsp;🐲</span>]]&nbsp;(&nbsp;[[User talk:Tenryuu|💬]]&nbsp;•&nbsp;[[Special:Contributions/Tenryuu|📝]]&nbsp;) 04:37, 23 May 2024 (UTC)
:::Unfortunately, the articles are drafted because they are not presently in an acceptable state for the encyclopedia. You are asking for help, but the criticism you describe is other editors trying to help you. [[User:Remsense|<span style="border-radius:2px 0 0 2px;padding:3px;background:#1E816F;color:#fff">'''Remsense'''</span>]][[User talk:Remsense|<span lang="zh" style="border:1px solid #1E816F;border-radius:0 2px 2px 0;padding:1px 3px;color:#000">诉</span>]] 04:41, 23 May 2024 (UTC)
:::@Briannemartindale. You keep refering to these things as {{tq|my articles}}, but there's really nothing about them that's yours per se as explained in [[:WP:OWN]]. Whatever you post on Wikipedia is only 100% yours up until you click the "Publish changes" button; once you do that you're irrevocably agreeing to allow others to edit it, even perhaps in ways that you mightn't like. The fact that the content we edit and create isn't really ours is probably one of the hardest things to get used to on Wikipedia, but it's something that we all have to come to terms with. The only thing that really matters is whether our edits are in accordance with relevant Wikipedia policies and guidelines. If we want more control and freedom, then we probably should look for [[:WP:ALTERNATIVE]]s that allow us to create what we want to create when we want to create. The hope is that through collaborative editing and adherence to relevant Wikipedia policies and guidelines, [[:WP:IMPERFECT|articles will be slowly improved over time]]; an article does, however, need a foundation to build on, and this foundation is Wikipedia notability. So, [[:WP:OVERCOME|it matters not how well written an article is if its foundation is found to be lacking]]. Draftifying articles is an alternative to nominating an article for deletion; it recognizes there might be some potential for a future article but its foundation is just too wobbly at the moment to survive an [[:WP:AFD]] discussion. Draftifying gives those interested a chance to strengthen the foundation and more clearly establish Wikipedia notability. As I mentioned in [[:WP:THQ#spelling and grammar help|another of your Teahouse questions]], creating one or two sentences stubs with questionable sourcing, no clear claim of Wikipedia notabilty and then asking others to fill in the gaps (i.e. establish the subject's Wikipedia notability) on the article's talk is not a good approach to take when trying to create articles; so, unless you slow down a bit and change your tack, you're likely going to find the articles you're creating starting to end up being nominated for deletion instead of being draftified. -- [[User:Marchjuly|Marchjuly]] ([[User talk:Marchjuly|talk]]) 07:09, 23 May 2024 (UTC)

== Unable to resubmit ==

I am getting an error @[[User:Theroadislong|Theroadislong]]when trhying to resubmit this with the requested changes. Any tips?

I am getting this error message: No stashed content found for (followed by a nonsensical arrangement of letters and dashes and numbers) [[User:Saraalutz|Saraalutz]] ([[User talk:Saraalutz|talk]]) 06:01, 23 May 2024 (UTC)

:Previous people who helped with this error said that it might be caused by having the edit page open for a very long time before publishing the changes, that you might need to copy your changes (if they aren't lost), click edit again, paste your changes and then try again. &ndash; [[Special:Contributions/2804:F14:80E4:8401:DCFE:5436:C21:470C|2804:F14:80E4:8401:DCFE:5436:C21:470C]] ([[User talk:2804:F14:80E4:8401:DCFE:5436:C21:470C|talk]]) 06:10, 23 May 2024 (UTC)
::I can only suggest you try again, your draft [[Draft:Debbie Matthews]] is VERY poorly sourced and will not be acceptable without better referencing. [[User:Theroadislong|Theroadislong]] ([[User talk:Theroadislong|talk]]) 06:33, 23 May 2024 (UTC)
:::in what way is the draft poorly sourced? can you be more specific please [[User:Saraalutz|Saraalutz]] ([[User talk:Saraalutz|talk]]) 06:36, 23 May 2024 (UTC)
::::Most of the draft cites no sources at all. Where did you get all that infomation from? Only the list of "Media Appearances:" cites sources. [[User:Maproom|Maproom]] ([[User talk:Maproom|talk]]) 07:02, 23 May 2024 (UTC)
:::::{{U|Saraalutz}} The first 30+ paragraphs are totally unsourced. [[User:Theroadislong|Theroadislong]] ([[User talk:Theroadislong|talk]]) 07:17, 23 May 2024 (UTC)
::::@[[User:Saraalutz|Saraalutz]] I believe you may have an undeclared 'conflict of interest' in writing about Debbie Matthews. Please follow the instructions and declare any connection you have with her on your userpage. See [[WP:COI]] for how to do this. If you are being paid in any way, you are obliged to declare who is paying you. Again, please read and follow [[WP:PAID]] to ensure you remain within our policy requirements whilst editing.
::::I am concerned that the large number of images [https://commons.wikimedia.org/wiki/Special:Contributions/Saraalutz you have uploaded to Commons] suggests you have direct personal access to photographs collated by Debbie Matthews, and that you do not understand the way Wikimedia Commons works. I would point out that even if Debbie Matthews holds those photos, she will not own the copyright to many of them - especially those taken whilst she was racing on her bike and not holding the camera! Unless you were the photographer, yourself, you will not have the legal right to release another person's photos under a Creative Commons licence for anyone else to use. Equally, whilst we encourage you to cite news stories from newspapers in which she is mentioned, you may not upload photos of newspaper pages to Commons as they remain copyright of the newspaper publisher. You do not have the rights to release them, either. Do not be surprised if many of these photos are marked for future deletion. Kind regards, [[User:Nick Moyes|Nick Moyes]] ([[User talk:Nick Moyes|talk]]) 08:29, 23 May 2024 (UTC)
:::::{{u|Saraalutz}}, there is no way under the sun that such a poorly referenced draft can be accepted into the encyclopedia. You have provided no way for readers to verify that many, many claims in your draft are true. Please be aware that [[WP:V|Verifiability]] is a core content policy, as is [[WP:OR|No original research]] which is also applicable. [[User:Cullen328|Cullen328]] ([[User talk:Cullen328|talk]]) 08:31, 23 May 2024 (UTC)
:::::I declared my COI in my userpage. Where else do I need to declare it? [[User:Saraalutz|Saraalutz]] ([[User talk:Saraalutz|talk]]) 07:32, 25 May 2024 (UTC)
::::@[[User:Saraalutz|Saraalutz]] I strongly recommend that you read [[Wikipedia:Writing Wikipedia articles backward]]. [[User:Shantavira|Shantavira]]|[[User talk:Shantavira|<sup>feed me</sup>]] 08:34, 23 May 2024 (UTC)
:::::Can you review it again and tell me what *specifically* the draft: Debbie Matthews still needs? @[[User:Grabup|Grabup]] [[User:Saraalutz|Saraalutz]] ([[User talk:Saraalutz|talk]]) 07:27, 25 May 2024 (UTC)
::::::@[[User:Saraalutz|Saraalutz]]: Replied to my talk page. [[User:Grabup|<span style="color:blue;">Grab</span><span style="color:red; font-size:larger;">Up</span>]] - [[User talk:Grabup|<span style="color:green;">Talk</span>]] 07:41, 25 May 2024 (UTC)
:::::::You have some neck charging for your poor editing! You are being paid by the subject to write this, do your client a favour and learn how Wikipedia actually works before submitting this again. [[User:Theroadislong|Theroadislong]] ([[User talk:Theroadislong|talk]]) 07:55, 25 May 2024 (UTC)

== Editing ==

{{cot|Asked and answered at the Help desk}}
Hi!
I tried to edit the site of Avri Levitan, because everything that is noted there sounds like AI and he wanted me to change that. But everytime I edit something, it is gone the next day. May somebody help me with that? [[User:Musethica.as|Musethica.as]] ([[User talk:Musethica.as|talk]]) 08:50, 23 May 2024 (UTC)
{{cob}}

:You are a suckpuppet of [[User:Musethica]] blocked by {{ping|Cabayi}}? [[User:Grabup|<span style="color:blue;">Grab</span><span style="color:red; font-size:larger;">Up</span>]] - [[User talk:Grabup|<span style="color:green;">Talk</span>]] 08:54, 23 May 2024 (UTC)
:{{Courtesy link|Avri Levitan}} <span style="font-family:monospace;">'''<nowiki>'''[[</nowiki>[[User:CanonNi]]<nowiki>]]'''</nowiki>'''</span> ([[User talk:CanonNi|talk]] • [[Special:Contributions/CanonNi|contribs]]) 08:55, 23 May 2024 (UTC)

== "We need secondary sources not primary sources" ==

For the draft [[Draft:Kirchhoff-Clausius's Law]].
I don't understand, I have cited first secondary sources from Max Planck and others, then at the end primary sources, because primary sources are the origine of the Kirchhoff-Clausius Law. [[User:Malypaet|Malypaet]] ([[User talk:Malypaet|talk]]) 09:18, 23 May 2024 (UTC)

:@[[User:Malypaet|Malypaet]] Can I comment that our encyclopaedia expects people to write introductory 'lead' paragraphs in Plain English, and should not expect them to dive straight into formulae? Whilst the concepts may be over my head, I would expect you to write an introductory statement something along the lines of ''"XXXs law is a scientific observation that shows a relationship between xxx, yyyy and zzz. It is defined nowadays as "insert quotation". The law was named after xxx and yyy, who published their initial findings in year x. The law was so-named by Max Planck in year Y. K-C's Law has significance in the field of xxxx"'' Only then would I expect the scientific explanation and equations that follow.
:I would point out that the translated citation you linked to does not verify that it was named by Planck - only that he refers to it as "Now according to the well-known Kirchoff-Clausius law..." Does that imply he actually named the Law, or was it already well-known and referred to elsewhere? Or are you inferring that he named it thus? A quick word search of Planck's German article did not reveal mention of either Clausius or Kirchhof's names in the text (but I might have missed it). I suspect the topic may well prove to be notable, but I feel your draft needs restructuring to make it understandable, and the sources showing notability more apparent. You note on your talk page that {{tq|"Publications on this topic are rare"}}, so it may indeed be that it does not require its own article here. But you could at least ensure there is, at least, a reference to their publications in the relevant scientists' biographies. Regards, [[User:Nick Moyes|Nick Moyes]] ([[User talk:Nick Moyes|talk]]) 11:34, 23 May 2024 (UTC)
::Thanks, I will use your model. For the 1901 German version the search does not work, it is also in chapter II "''Nun ist nach dem bekannten Kirchhoff- Clausius 'schen
::Gesetz die von einer schwarzen Flache pro Zeiteinheit in ein
::...". In their biographies the books that demonstrate the law are cited, but in them it is just a formula demonstrated in one of the many chapters, not named as a law. These are two famous manuscripts listing all their work. Without this law, Max Planck would never have found justification for his law found empirically. [[User:Malypaet|Malypaet]] ([[User talk:Malypaet|talk]]) 19:55, 23 May 2024 (UTC)
::I support what {{u|Nick Moyes}} has said, and would add that the formulae are anyway of little use without an explanation of what c, f, λ etc. represent. [[User:Maproom|Maproom]] ([[User talk:Maproom|talk]]) 06:29, 24 May 2024 (UTC)

== "Unambiguous advertising and promotion" ==

Hi Teahouse!

I need some help in identifying which areas sounds like advertising and promotion for [[Draft:EQuest Education Group|this draft]]. I have read it multiple times but I can't identify where it sounds like an advertisement.

Thanks! [[User:Shengyongchoo|Shengyongchoo]] ([[User talk:Shengyongchoo|talk]]) 10:20, 23 May 2024 (UTC)

:Hi @[[User:Shengyongchoo|Shengyongchoo]]: it's promotional, because it's mostly the organisation telling the world about itself and what it does, which is pretty much the definition of promotion (see [[WP:YESPROMO]]). Wikipedia articles should instead be mainly based on what independent and reliable ''secondary'' sources have said about a subject and what makes it noteworthy. HTH, -- [[User:DoubleGrazing|DoubleGrazing]] ([[User talk:DoubleGrazing|talk]]) 11:01, 23 May 2024 (UTC)

::You start out by stating "According to EQuest Education Group..." Tell us what a newspaper or TV journalist has to say about the institution. Has EQuest or anyone on its staff won important awards? If so, write about that. EQuest probably wants to promote itself, so don't rely on data provided by EQuest. [[User:Karenthewriter|Karenthewriter]] ([[User talk:Karenthewriter|talk]]) 12:48, 23 May 2024 (UTC)
:Hello, Shengyongcnoo. Please note that {{HD/WINI}} [[User:ColinFine|ColinFine]] ([[User talk:ColinFine|talk]]) 17:17, 23 May 2024 (UTC)

== Proposing a page move ==

Hey there! Is there a way to '''propose''' a page move? I know you can request one, but is there a wau to propose one to the general community? Thanks! :D [[User:MemeGod27|MemeGod ._.]] <small>([[User talk:MemeGod27|My talk page]], [[Special:Contribs/MemeGod27|my contributions]] and [[User:MemeGod27/Creations|my creations!]])</small> 11:26, 23 May 2024 (UTC)
:[[WP:RM]] is the place to go. [[User:Geardona|Geardona]] ([[User talk:Geardona|talk to me?]]) 11:46, 23 May 2024 (UTC)
:@[[User:MemeGod27|MemeGod27]] If the move is at all controversial, you need to follow the guidance at [[WP:PCM]], which also defines "controversial" in this context. [[User:Michael D. Turnbull|Mike Turnbull]] ([[User talk:Michael D. Turnbull|talk]]) 13:36, 23 May 2024 (UTC)

== Gloucester image renewal ==
Im a WikiProject Gloucestershire user who lives in the city (by that i mean the [[Gloucester|ONLY CITY]]). Im planning on renewing images related to Gloucester if i can get my hands on a camera. Please send thanks if i can! <span style="color:#f1a8ff">{{#if:|{{#ifexpr:({{#time:U|{{{3}}}}} - {{#time:U|now}}) > 0|{{highlight/core|~ '''Snipertron12 :3''' ~|{{#ifeq:black||yellow|black}}}}|~ '''Snipertron12 :3''' ~}}|{{highlight/core|~ '''Snipertron12 :3''' ~|black}}}}</span> [|[[User:Snipertron12|User]]|[[User talk:Snipertron12|Talk]]|[[Special:Contributions/Snipertron12|Cont]]|] 12:56, 23 May 2024 (UTC)

:Hi @[[User:Snipertron12|Snipertron12]], welcome to the Teahouse. Is there a question you have about using or editing Wikipedia? Are you asking if you can get money from the Wikimedia Foundation to buy a camera? [[Special:Contributions/57.140.16.48|57.140.16.48]] ([[User talk:57.140.16.48|talk]]) 13:31, 23 May 2024 (UTC)
:Our article on [[Gloucester]] has several excellent images and probably doesn't need any more. Unless you can genuinely improve on those images, your contributions are likely to be removed. [[User:Shantavira|Shantavira]]|[[User talk:Shantavira|<sup>feed me</sup>]] 13:47, 23 May 2024 (UTC)
::While the main Gloucester article has enough images there are subtopics in [[:Category:Gloucester]] that are missing images, such as [[Gloucester Academy]]. [[User:MKFI|MKFI]] ([[User talk:MKFI|talk]]) 14:37, 23 May 2024 (UTC)
:::Skimming related articles, I noticed that there's no image for [[Meadow Park, Gloucester]] and that [[Gloucestershire Royal Hospital]] does not have a particuarly great quality image. I'm sure it would be much appreciated if {{u|Snipertron12}} created some high quality photos for those. -- [[User:DandelionAndBurdock|D'n'B]]-''[[User_talk:DandelionAndBurdock|t]]'' -- 06:33, 24 May 2024 (UTC)

:@[[User:Snipertron12|Snipertron12]] If you mean you want to change some of the pics at for example [[Gloucester]] to pics you have taken yourself, see [[WP:BOLD]]. There may be discussions. [[User:Gråbergs Gråa Sång|Gråbergs Gråa Sång]] ([[User talk:Gråbergs Gråa Sång|talk]]) 15:02, 23 May 2024 (UTC)
:I can't find it at the moment but somewhere on Wikipedia there is a tool that will give you a list of articles flagged with [[:Template:Photo requested]] that are within your local area. Perhaps someone else can provide a link. [[User:Shantavira|Shantavira]]|[[User talk:Shantavira|<sup>feed me</sup>]] 10:10, 24 May 2024 (UTC)

== Template:Main not working a specific article? ==

Hi! Not sure if the Teahouse is the correct place to ask this, but I'm trying to add the link [[4D N = 1 supergravity]] into Main:Template
link to [[Supergravity#4D N = 1 SUGRA]], but for some reason its not working. Other links seem to work but this one does not. The error is: {{Main|4D N = 1 supergravity}}

What am I doing wrong? I suspect it's something silly. Very best thanks! [[User:OpenScience709|OpenScience709]] ([[User talk:OpenScience709|talk]]) 14:09, 23 May 2024 (UTC)
:[[User:OpenScience709|OpenScience709]]: The = sign is the problem. Try <nowiki>{{Main|1=4D N = 1 supergravity}} </nowiki> —[[User:Kusma|Kusma]] ([[User talk:Kusma|talk]]) 14:15, 23 May 2024 (UTC)
::Yes! That works! Thanks!! [[User:OpenScience709|OpenScience709]] ([[User talk:OpenScience709|talk]]) 14:19, 23 May 2024 (UTC)

== Help citing on-screen information ==

Let’s say I want to cite the fact that a character in a cartoon TV show lived in a treehouse. I understand the best method is to cite a news article or press release advertising this, but I can’t find a source mentioning this specific fact.

In absence of this, what’s the best way to do it? Several episodes of the show display it on-screen. Can I cite one of the episodes or is that considered original research? [[User:Jaggywar|Jaggywar]] ([[User talk:Jaggywar|talk]]) 14:36, 23 May 2024 (UTC)

:@[[User:Jaggywar|Jaggywar]] Depends on the situation. If this is part of a plot section, or something like [[List_of_Two_and_a_Half_Men_characters#Jake_Harper]], [[MOS:PLOTSOURCE]] can be argued to apply. You can also make a cite like<ref>''Treehouse cartoon'', episode one season one</ref> [[User:Gråbergs Gråa Sång|Gråbergs Gråa Sång]] ([[User talk:Gråbergs Gråa Sång|talk]]) 15:09, 23 May 2024 (UTC)
{{reftalk}} [[User:Gråbergs Gråa Sång|Gråbergs Gråa Sång]] ([[User talk:Gråbergs Gråa Sång|talk]]) 15:09, 23 May 2024 (UTC)

== on more visual editor table jank ==

this is currently specifically regarding the last entry of the [[List of generation III Pokémon|list of gen 3 pokémon]] (that being deoxys), though it might be useful elsewhere in the future maybe

trying to merge cells with templates (such as the recently-ish created [[Template:Poketype|poketype]]) or adding templates that affect cell colors to already merged cells doesn't seem to work at all on the visual editor

am i missing something, or has that just not been ironed out yet? '''[[User:Cogsan|<span style="color:#8a440a">cogsan</span>]] <sub>[[User talk:Cogsan|<span style="color:#8a440a">(nag me)</span>]] [[Special:Contributions/Cogsan|<span style="color:#8a440a">(stalk me)</span>]]</sub>''' 16:31, 23 May 2024 (UTC)
: {{re|Cogsan}} [[Wikipedia:VisualEditor#Limitations]] says that VE does not work well with tables and some templates, such as the {{t|na}} template in use in the table you mention. [[User:RudolfRed|RudolfRed]] ([[User talk:RudolfRed|talk]]) 02:48, 24 May 2024 (UTC)
::d*ng, time to learn how tables work in the source editor (or leave it as is, it could be fine-ish) (or copy from [[rotom]] a gen later)
::thanks '''[[User:Cogsan|<span style="color:#8a440a">cogsan</span>]] <sub>[[User talk:Cogsan|<span style="color:#8a440a">(nag me)</span>]] [[Special:Contributions/Cogsan|<span style="color:#8a440a">(stalk me)</span>]]</sub>''' 11:12, 24 May 2024 (UTC)

== What happens next? ==

I just published an article and I have made several edits on it. I was wondering what the next step is to finding out if it is approved. Thank you!

[[User:Ryanpaynter1|Ryanpaynter1]] ([[User talk:Ryanpaynter1|talk]]) 17:13, 23 May 2024 (UTC)
:{{editconflict}} Hi {{u|Ryanpaynter1}} - you don't appear to have published an article. The only edits I can see are to your [[WP:USERPAGE|userpage]]. Perhaps you were trying to create a draft at [[WP:AfC|Articles for Creation]]. That said, if what you've [https://en.wikipedia.org/w/index.php?title=User:Ryanpaynter1&oldid=1225312376 currently got on userpage] were submitted, I can assure you that would not be approved. I'd take a look at [[WP:AUTOBIOGRAPHY]] for starters, and especially take note that user generated sources like IMDb and Wikipedia are never considered reliable. -- [[User:DandelionAndBurdock|D'n'B]]-''[[User_talk:DandelionAndBurdock|t]]'' -- 17:25, 23 May 2024 (UTC)


:Hello, Ryanpaynter1, and welcome to the Teahouse and to Wikipedia.
:I'm afraid that, like many people who come here for the purpose of promotion, you have fundamentally misunderstood what Wikipedia is.
:Please see [[WP:autobiography|autobiography]] to understand why writing about yourself on Wikipedia is very strongly discouraged.
:In addition, you have written your attempted autobiography on your [[WP:user page|user page]], which is not part of the encyclopaedia, and not an appropriate place to write an article.

:If there were any point, I would consider [[WP:MOVE|moving]] your draft to [[Draft:Ryan Paynter]], but I'm afriad that (like most new editors who attempt the challenging task of writing an article before they have learnt the necessary skills) you have written it [[WP:BACKWARDS|backwards]], and I doubt that much of it is salvageable. [[User:ColinFine|ColinFine]] ([[User talk:ColinFine|talk]]) 17:23, 23 May 2024 (UTC)
::Colin, thank you so much for responding. I am new to this and wasn’t sure how to proceed. I am mentioned in articles o. This site, but wasn’t sure how to be tagged (blue). Should I delete the article? Or put it in a different location? Thank you for your advice and time. [[Special:Contributions/2603:6082:CF00:80:FC63:6013:95E:A928|2603:6082:CF00:80:FC63:6013:95E:A928]] ([[User talk:2603:6082:CF00:80:FC63:6013:95E:A928|talk]]) 17:28, 23 May 2024 (UTC)
:::If, after looking at all the links we have given, you want to proceed, please read [[WP:your first article|your first article]]. Note that, even where there is not a [[WP:conflict of interest|conflict of interest]], I invariably advise any new editor to spend at least several weeks making edits to existing articles and learning about core policies such as [[WP:verifiability|verifiability]], [[WP:notability|notability]], [[WP:reliable sources|reliable sources]], and [[WP:neutral point of view|neutral point of view]] before even trying to create an article. [[User:ColinFine|ColinFine]] ([[User talk:ColinFine|talk]]) 17:34, 23 May 2024 (UTC)
:As a further reply, expanding on what DandelionandBurdock and I have said: If you meet Wikipedia's criteria for [[WP:notability|notability]] - roughly, that there is enough reliably published material (not user-generated sites such as iMDB and Wikipedia) that is wholly independent of you (not written, published, commissioned, or based on the words of, you or your associates) to base an article on - then there could be an article about you. As we have both said, you are strongly discouraged from writing it yourself.
:Whoever wrote such an article, it would not belong to you, would not be controlled by you, may be edited by almost anybody in the world ''except'' you and your associates, and should be based almost entirely on what those independent sources said about you, not on what you or your associates say or want to say. Please see [[WP:an article about yourself isn't necessarily a good thing|an article about yourself isn't necessarily a good thing]]. [[User:ColinFine|ColinFine]] ([[User talk:ColinFine|talk]]) 17:31, 23 May 2024 (UTC)
::Thank you for the information. I was attempting to link my name to the Deadline articles and movie article as some of the other fellow actors are linked. I will delete and let the process play out. I appreciate your input. Best. [[User:Ryanpaynter1|Ryanpaynter1]] ([[User talk:Ryanpaynter1|talk]]) 17:52, 23 May 2024 (UTC)

== Beastie Boys Square ==

I am curious as to why [[Beastie Boys Square]] would be merged with Paul's Boutique when there are hundreds of articles dating back 15 years... Doesn't seem appropriate to merge a destination in NYC to an album when there is justification and a story. [[User:VeniceBreeze|VeniceBreeze]] ([[User talk:VeniceBreeze|talk]]) 18:45, 23 May 2024 (UTC)
:{{courtesy link|WP:Articles for deletion/Beastie Boys Square}} --[[User:ColinFine|ColinFine]] ([[User talk:ColinFine|talk]]) 19:41, 23 May 2024 (UTC)

:This feels like inappropriate [[WP:CANVASS]]ing. Your own take can be kept to the discussion page itself. -- [[User:DandelionAndBurdock|D'n'B]]-''[[User_talk:DandelionAndBurdock|t]]'' -- 20:27, 23 May 2024 (UTC)

== Question about comments ==

Hey Teahouse, I'm wondering if you can add comments when you're editing, not like one of those boxes at the bottom when you're editing which is called an edit summary. Is there some kind of text that you need or some special character before you start typing your 'comment'? [[User:GoodHue291|GoodHue291]] ([[User talk:GoodHue291|talk]]) 19:09, 23 May 2024 (UTC)

:HI, GoodHue291. If you mean a comment that is visible only to somebody who is editing the page, see [[WP:COMMENT]]. [[User:ColinFine|ColinFine]] ([[User talk:ColinFine|talk]]) 19:42, 23 May 2024 (UTC)
::Yeah this is what I was talking about. [[User:GoodHue291|GoodHue291]] ([[User talk:GoodHue291|talk]]) 20:26, 23 May 2024 (UTC)

:{{re|GoodHue291}} If you need to add an explanatory note for readers, please see [[H:NOTES]]. --[[User:CiaPan|CiaPan]] ([[User talk:CiaPan|talk]]) 19:58, 23 May 2024 (UTC)

:OTOH, if you mean a comment with a request to other editors, like this one: [[special:diff/1225329855]], then the best option is to put it plainly visible at the respective article's Talk page. In that case it would be [[Talk:Sishui Township]]. Please see [[H:TALK]] for more info. --[[User:CiaPan|CiaPan]] ([[User talk:CiaPan|talk]]) 20:10, 23 May 2024 (UTC)

== Daajing Giids, B.C. Canada ==

In the first paragraph of the towns description there is an error in its location. It is not located anywhere near Skidegate inlet. They are about 30 Km apart. Try looking up Skidegate inlet on a marine map. Skidegate village, Skidegate lake , and Skidegate inlet are not located anywhere close together. Daajing Giids is located in Bear Skin Bay not Skidegate inlet. Skidegate inlet is located on the west side of Gram island. Can I get some help to fix this please? [[Special:Contributions/209.53.84.49|209.53.84.49]] ([[User talk:209.53.84.49|talk]]) 20:50, 23 May 2024 (UTC)
:{{courtesy link|Daajing Giids}}
:I note that the map in the [https://geonames.nrcan.gc.ca/search-place-names/unique?id=JDMYG CGNDB entry] for the village does place it on Skidegate Inlet. [[User:Deor|Deor]] ([[User talk:Deor|talk]]) 21:11, 23 May 2024 (UTC)
: Hi IP 209.53.84.49. Have you tried discussing this at [[:Talk:Daajing Giids]]? Generally, the best place to discuss article content is on its corresponding article talk page because that's typically those most interested and familiar with the subject matter will be best able to respond. Article talk page discussion almost makes it much easier to keep all discussion relevant to the article in one place for archiving and reference purposes. As for {{tq|Try looking up Skidegate inlet on a marine map}}, generally that's not the best way to approach something like this; it's much more helpful for you yourself to actually provide a link to a [[:WP:RS|reliable source]] supporting the changes you feel need to be made instead of just saying to to others "look it up". You should also understand that a map is likely to be considered to be a [[:WP:PRIMARY]] source and this limits how they can be cited as a reference for Wikipedia's purposes. Wikipedia policy doesn't really allow use to interpret such a map ourselves, but we can cite interpretations made by [[:WP:SECONDARY]] reliable sources about the map and about the location of Daajing Giids. -- [[User:Marchjuly|Marchjuly]] ([[User talk:Marchjuly|talk]]) 21:28, 23 May 2024 (UTC)

== Rejected Article ==

Hi Teahouse, my article [[Draft:Wancewot|Wancewot]] was just rejected because there were not enough reliable sources. The problem with this is that I wrote this article after hearing about the legend on my trip to Poland, and no one else had wrote about it! Other than one or two amateur websites made by locals, there are literally no sources to link to. When I heard this legend, I wanted to share it with everyone else because it is one of the myths the town of [[Wancerzów]] is based on, and the legend itself is very intriguing. Do you have any ways that I can get this through review? [[User:Blackwell09|Blackwell09]] ([[User talk:Blackwell09|talk]]) 21:16, 23 May 2024 (UTC)

:Hi @[[User:Blackwell09|Blackwell09]], welcome to the Teahouse. If it's true that {{tq|Other than one or two amateur websites made by locals, there are literally no sources to link to}}, then unfortunately there is no way to get this through review. Wikipedia exists to summarize what has already been published in [[WP:reliable sources|reliable sources]], not to ''be'' the source publishing something or "getting the word out". It might help to read [[WP:42]]. [[Special:Contributions/57.140.16.48|57.140.16.48]] ([[User talk:57.140.16.48|talk]]) 21:22, 23 May 2024 (UTC)
::So would you just suggest creating a website to tell others, and encourage them to visit Poland to solidify the sources? [[User:Blackwell09|Blackwell09]] ([[User talk:Blackwell09|talk]]) 21:33, 23 May 2024 (UTC)
:::@[[User:Blackwell09|Blackwell09]] Our advice is that this has, currently, no place on Wikipedia, and that another amateur website, ten other amateur websites, will not make it have a place.
:::We require references from ''significant'' coverage ''about'' the topic of the article, and ''independent'' of it, in ''multiple [[WP:SECONDARY|secondary sources]]'' which are ''[[WP:RS]]'' please. See [[WP:42]]. Please also see [[WP:PRIMARY]] which details the limited permitted usage of primary sources and [[WP:SELFPUB]] which has clear limitations on self published sources. Providing sufficient references, ideally one per fact referred to, that meet these tough criteria is likely to allow this article to remain. Lack of them or an inability to find them is likely to mean that the topic is not suitable for inclusion, certainly today. 🇺🇦&nbsp;[[User:Timtrent|<span style="color:#800">Fiddle</span><sup><small>Timtrent</small></sup>]]&nbsp;[[User talk:Timtrent|<span style="color:#070">Faddle</span><sup><small>Talk&nbsp;to&nbsp;me</small></sup>]]&nbsp;🇺🇦 21:44, 23 May 2024 (UTC)
:::@[[User:Blackwell09|Blackwell09]], have you heard of [[Fandom (website)|Fandom]]? That is a collection of wikis on various subjects, and their inclusion criteria are much different from Wikipedia's. There is, specifically, a Myth and Folklore Wiki where you might be able to create an article, and there may be similar wikis on similar subjects over on Fandom; I'd recommend checking those out. [[Special:Contributions/57.140.16.48|57.140.16.48]] ([[User talk:57.140.16.48|talk]]) 22:04, 23 May 2024 (UTC)
::::Ok, thank you for all the advice! Yes, I will definitely post to Fandom, thank you for the suggestion! [[User:Blackwell09|Blackwell09]] ([[User talk:Blackwell09|talk]]) 00:36, 24 May 2024 (UTC)

== [[Bereznianka]] ==

This stub is almost identical to the content of reference #1, https://www.danceanddance.com/207/Dance_styles_review.php. I can't tell if the Wikipedia article is a copyright violation, or if the danceanddance page is a copy of the Wikipedia article. [[Special:Contributions/76.14.122.5|76.14.122.5]] ([[User talk:76.14.122.5|talk]]) 22:47, 23 May 2024 (UTC)

:Okay, looking at the history, I'm inclined to say that the danceanddance page copied ours, then somebody (seeing our info was unreferenced), Googled the topic and cited the first website that popped up. I'll double check, but I remove the reference (because we don't cite Wikipedia mirrors) and put a message on the talk page. [[User:GreenLipstickLesbian|GreenLipstickLesbian]] ([[User talk:GreenLipstickLesbian|talk]]) 23:25, 23 May 2024 (UTC)
::Thank you. [[Special:Contributions/76.14.122.5|76.14.122.5]] ([[User talk:76.14.122.5|talk]]) 01:55, 24 May 2024 (UTC)

== how to create a page ==

how to create a page [[Special:Contributions/95.92.143.182|95.92.143.182]] ([[User talk:95.92.143.182|talk]]) 00:36, 24 May 2024 (UTC)

:How to respond? First, [[Help:Introduction|learn how to edit]]. Once you're proficient at that (and not before), read about creating [[Help:Your first article|Your first article]]. -- [[User:Hoary|Hoary]] ([[User talk:Hoary|talk]]) 00:55, 24 May 2024 (UTC)

== Wikipedia Commons Image Deletion ==

Hi,

I'll get right into it. About a year and a half ago, I took a rather bad photo of my grandfather, edited it using faceapp to give him a goofy, horrible, hairstyle and then inserted that image into his Wikipedia page. It was only for a joke, I just screenshotted it to send to him then removed it a few minutes later. But then, just earlier today, he texted me that it was somehow still on his Wikipedia page. I looked through the edit history and apparently someone had added clearly joke image back in March. I removed that image once again and replaced it with a better one, but the image still pops up when you search his name on google.

He just wrote a book and I feel horrible since that image is now online because it's in the Wikipedia commons. I'm not the best with tech so I had no idea that everyone would be able to see and upload that image. Can you please help me fix this and get that image off the Wikipedia commons? I submitted a removal request but as I said I'm not great with tech, I don't know if it went through. I also need to make sure it gets deleted. I don't want this to damage his professional career. I already feel horrible.
Here is the link to the Wikipedia commons page with the image: https://commons.wikimedia.org/wiki/File:John_Darnton_2022_restaurant.jpg
Here is the link to his Wikipedia page: https://en.wikipedia.org/wiki/John_Darnton

Please do what you can, I'm in a pretty desperate situation here.

Thanks,
Joe [[User:JohnDoe887|JohnDoe887]] ([[User talk:JohnDoe887|talk]]) 01:20, 24 May 2024 (UTC)

:Hello, @[[User:JohnDoe887|JohnDoe887]] and welcome to the Teahouse. Images uploaded to Commons has to be dealt on commons, we have no say over the other projects. Wikipedia and Wikimedia Commons go hand in hand but editors here sometimes don’t edit there. I saw that it was nominated for deletion on Commons so you can wait to see what the solution is over there. I also ask that you do not edit your grandfather’s wikipedia page as this is a [[WP:COI|conflict of interest]] in the eyes of the community. Thank you, <span style="font-family:Arial;background-color:#fff;border:2px dashed#69c73e">[[User:Cowboygilbert|<span style="color:#3f6b39">'''Cowboygilbert'''</span>]] - [[User talk:Cowboygilbert|<span style="color:#d12667"> (talk) ♥</span>]]</span> 01:28, 24 May 2024 (UTC)
::Ok thank you I was worried it didn't go through. [[User:JohnDoe887|JohnDoe887]] ([[User talk:JohnDoe887|talk]]) 01:29, 24 May 2024 (UTC)

== Do not have the permissions to move page from Sandbox to Draft Namespace for review ==

Hello, I have created an article in my sandbox ([[User:Dr_Abhishek_Yaadav/sandbox]]) and would like to move it to the Draft namespace as "Draft:Dr Abhishek Yadav" for review. However, I am not autoconfirmed and do not have the permissions to move pages. Could someone please assist me with this move? Thank you! [[User:Dr Abhishek Yaadav|Dr Abhishek Yaadav]] ([[User talk:Dr Abhishek Yaadav|talk]]) 06:07, 24 May 2024 (UTC)

:You are writing about your self? [[User:Grabup|<span style="color:blue;">Grab</span><span style="color:red; font-size:larger;">Up</span>]] - [[User talk:Grabup|<span style="color:green;">Talk</span>]] 06:11, 24 May 2024 (UTC)
::{{u|Dr Abhishek Yaadav}}, your sandox page has been nominated for speedy deletion. Self-promotion is not permitted on Wikipedia. Please read [[WP:AUTOBIOGRAPHY]]. [[User:Cullen328|Cullen328]] ([[User talk:Cullen328|talk]]) 06:14, 24 May 2024 (UTC)
:::I got it, Now, please confirm the below:
:::The process I followed: I have written content for artcle. I have created a new account. Clicked on sandbox. Drafted Article. Submitted for review. Its got rejected due to self promoting.
:::Please confirm:
:::1. Did I follow the correct process? Or something I missed?
:::2. Can I mention these words in article
:::Liver Surgeon in Pune
:::Liver Transplant in Pune
:::Liver Doctor in Pune
:::Liver Surgery Doctor in Pune
:::I think it might look like promotional?
:::3. Is there any specific guideline related to account age?
:::4. Or any suggestions to post a bio-graphic article for others. [[User:Dr Abhishek Yaadav|Dr Abhishek Yaadav]] ([[User talk:Dr Abhishek Yaadav|talk]]) 07:34, 24 May 2024 (UTC)
::::@[[User:Dr Abhishek Yaadav|Dr Abhishek Yaadav]]: please read and understand [[WP:AUTOBIO]]. TL;DNR = you shouldn't be writing about yourself in the first place. You may want to try LinkedIn etc. -- [[User:DoubleGrazing|DoubleGrazing]] ([[User talk:DoubleGrazing|talk]]) 07:36, 24 May 2024 (UTC)
::::Please read the [[WP:AUTO|autobiography policy]]; you really shouldn't be writing about yourself at all. Wikipedia is not a place for people to tell about their professional qualifications and services they offer. Please use social media to do that. Wikipedia is a place for independent editors to write about topics that they notice receive significant coverage in independent [[WP:RS|reliable sources]] and are [[WP:N|notable as Wikipedia uses the word]]. Wikipedia wants to know what others say about a topic, not what it says about itself. [[User:331dot|331dot]] ([[User talk:331dot|talk]]) 07:37, 24 May 2024 (UTC)
::::@[[User:Dr Abhishek Yaadav|Dr Abhishek Yaadav]]: can I also add that registering a new account and submitting the same draft from that does not make it any more acceptable. See also [[WP:SOCK]]. -- [[User:DoubleGrazing|DoubleGrazing]] ([[User talk:DoubleGrazing|talk]]) 08:56, 24 May 2024 (UTC)

This account indefinitely blocked. [[User:David notMD|David notMD]] ([[User talk:David notMD|talk]]) 11:07, 25 May 2024 (UTC)

== Can anyone approve this draft? ==

There is a draft that I can't resubmit; [[Draft:Fantastic Beasts: The Secrets of Dumbledore (soundtrack)]]; it looks like it is protected [[Special:Contributions/201.188.149.181|201.188.149.181]] ([[User talk:201.188.149.181|talk]]) 08:20, 24 May 2024 (UTC)

:Yes, it's semi-protected, and for good reason. It didn't merit promotion to article status when {{U|Bilorv}} declined it on 1 April; and since then it has been resubmitted several times by some Chilean IP ''without improvement''. Now you (a Chilean IP) perhaps want somebody to resubmit it yet again ''without improvement''. Which part of the repeated "declined" template, or which comment, do you not understand? -- [[User:Hoary|Hoary]] ([[User talk:Hoary|talk]]) 08:51, 24 May 2024 (UTC)
::Block evasion IP yet again. <span style="solid;background:#a3b18a; border-radius: 4px; -moz-border-radius: 4px; font-family: Papyrus">'''[[User:MikeAllen|<span style="color: #606c38">Mike</span>]]&nbsp;[[User talk:MikeAllen|<span style="color:#606c38">Allen</span>]]'''</span> 16:19, 24 May 2024 (UTC)
::That is not the point, the thing is that it is protected and it has everything to be approved; such as [[Fantastic Beasts and Where to Find Them (soundtrack)]] and [[Fantastic Beasts: The Crimes of Grindelwald (soundtrack)]] [[Special:Contributions/201.188.149.181|201.188.149.181]] ([[User talk:201.188.149.181|talk]]) 21:25, 24 May 2024 (UTC)
:::You are not supposed to be editing Wikipedia since your other [[Special:Contributions/201.188.133.126|IPs are blocked]]. No one is going to take you seriously. That's the point. <span style="solid;background:#a3b18a; border-radius: 4px; -moz-border-radius: 4px; font-family: Papyrus">'''[[User:MikeAllen|<span style="color: #606c38">Mike</span>]]&nbsp;[[User talk:MikeAllen|<span style="color:#606c38">Allen</span>]]'''</span> 22:12, 24 May 2024 (UTC)

== Manhattan map issue ==
For some reason, Manhattan locations on the maps are shown in various balkan languages. Is this a bug? Please help! <span style="color:#f1a8ff">{{#if:|{{#ifexpr:({{#time:U|{{{3}}}}} - {{#time:U|now}}) > 0|{{highlight/core|~ '''Snipertron12 :3''' ~|{{#ifeq:black||yellow|black}}}}|~ '''Snipertron12 :3''' ~}}|{{highlight/core|~ '''Snipertron12 :3''' ~|black}}}}</span> [|[[User:Snipertron12|User]]|[[User talk:Snipertron12|Talk]]|[[Special:Contributions/Snipertron12|Cont]]|] 09:24, 24 May 2024 (UTC)
:{{ping|Snipertron12}} Which map please? The ones I looked at at [[Manhattan]] all looked ok. [[User:Lectonar|Lectonar]] ([[User talk:Lectonar|talk]]) 09:45, 24 May 2024 (UTC)
::: [[Eldridge_Street_Synagogue#/map/0|This one]]. <span style="color:#f1a8ff">{{#if:|{{#ifexpr:({{#time:U|{{{3}}}}} - {{#time:U|now}}) > 0|{{highlight/core|~ '''Snipertron12 :3''' ~|{{#ifeq:black||yellow|black}}}}|~ '''Snipertron12 :3''' ~}}|{{highlight/core|~ '''Snipertron12 :3''' ~|black}}}}</span> [|[[User:Snipertron12|User]]|[[User talk:Snipertron12|Talk]]|[[Special:Contributions/Snipertron12|Cont]]|] 09:49, 24 May 2024 (UTC)
::::This comes in automagically via [[Openstreetmaps]]. As the page already has a map for its location in Lower Manhattan, you could ask [[User:Epicgenius|Epicgenius]], who added the OSM link, if it is really necessary to have somewhat redundant maps; other locations do not necessarily have them. [[User:Lectonar|Lectonar]] ([[User talk:Lectonar|talk]]) 10:33, 24 May 2024 (UTC)
:::::Its not just that building, its the entierty of Manhattan. <span style="color:#f1a8ff">{{#if:|{{#ifexpr:({{#time:U|{{{3}}}}} - {{#time:U|now}}) > 0|{{highlight/core|~ '''Snipertron12 :3''' ~|{{#ifeq:black||yellow|black}}}}|~ '''Snipertron12 :3''' ~}}|{{highlight/core|~ '''Snipertron12 :3''' ~|black}}}}</span> [|[[User:Snipertron12|User]]|[[User talk:Snipertron12|Talk]]|[[Special:Contributions/Snipertron12|Cont]]|] 11:58, 24 May 2024 (UTC)
::::::{{ping|Snipertron12}}, see [[Talk:Eldridge Street Synagogue#New detail map of Lower East Side]]. Epicgenius is aware of the issue and it has been raised at [[Wikipedia:Village pump (technical)]] and [[Wikipedia:Phabricator]]. It seems to be a bug which is not easy to fix. [[User:TSventon|TSventon]] ([[User talk:TSventon|talk]]) 13:37, 24 May 2024 (UTC)

==re-reverting==
I tried to re-revert the undo that was done here https://en.wikipedia.org/w/index.php?title=British_currency_in_the_Middle_East&diff=prev&oldid=1218650667 in the mistaken belief that the IP server was a sockpuppet. But all I got was a page explaining about reverting. I'd be grateful if anybody could help in this regard. [[User:Specialrequestaccount|Specialrequestaccount]] ([[User talk:Specialrequestaccount|talk]]) 10:52, 24 May 2024 (UTC)

:@[[User:Specialrequestaccount|Specialrequestaccount]] I don't know exactly what has been going on but I see there is a comment on the Talk Page at [[Talk:British currency in the Middle East]] by [[User:JMF]], who is a very experienced editor. I suggest you continue his discussion there to come to a [[WP:CONSENSUS|consensus]] as to what should be done next. [[User:Michael D. Turnbull|Mike Turnbull]] ([[User talk:Michael D. Turnbull|talk]]) 18:37, 24 May 2024 (UTC)
::.... see also [[WT:WikiProject Numismatics#TheCurrencyGuy Sock Edits]]. [[User:Michael D. Turnbull|Mike Turnbull]] ([[User talk:Michael D. Turnbull|talk]]) 21:22, 24 May 2024 (UTC)
::@[[User:Specialrequestaccount|Specialrequestaccount]]: Please first consider carefully what Mike Turnbull has said, but on the specific issue, did you perhaps click on the tag, i.e. <code>([[Special:Tags|Tag]]'': [[Wikipedia:Undo|Undo]]'')</code>, instead of clicking undo at the top, i.e. <code>'''{{color|#36c|Revision as of 23:59, 12 April 2024}} ({{color|#36c|edit}}) ({{color|#36c|undo}})'''</code>?
::The tag, of course, leads to a page describing what an undo is, while the undo at the top takes you to a page for undoing the edit (if there haven't been any other edits afterwards that would conflict with the revert, if there is conflict there will be an error, as is the case with the revision you've linked). &ndash; [[Special:Contributions/2804:F14:8085:6201:EC0A:3D07:F537:12EF|2804:F14:8085:6201:EC0A:3D07:F537:12EF]] ([[User talk:2804:F14:8085:6201:EC0A:3D07:F537:12EF|talk]]) 22:34, 24 May 2024 (UTC)
Thanks for your help. Meanwhile, I sorted it another way by using the earlier template. But the information which you have given me above should be useful generally for the future. [[User:Specialrequestaccount|Specialrequestaccount]] ([[User talk:Specialrequestaccount|talk]]) 23:09, 24 May 2024 (UTC)

== Wikipedians ==

Who can be considered the greatest wikipedian of all time?
And the worst vandal/troll? [[User:Calicanto2023|Calicanto2023]] ([[User talk:Calicanto2023|talk]]) 12:40, 24 May 2024 (UTC)

:[[Jimmy Wales]] ''can'' be considered to be the greatest Wikipedian of all time. Or one of the editors at the top of [[Wikipedia:List of Wikipedians by number of edits]]. Or someone else. It depends on the considerer. [[User:Gråbergs Gråa Sång|Gråbergs Gråa Sång]] ([[User talk:Gråbergs Gråa Sång|talk]]) 12:52, 24 May 2024 (UTC)
:@[[User:Calicanto2023|Calicanto2023]] Wait... aren't you an alternate account of {{noping|14 novembre}}? <span style="font-family:monospace;">'''<nowiki>'''[[</nowiki>[[User:CanonNi]]<nowiki>]]'''</nowiki>'''</span> ([[User talk:CanonNi|talk]] • [[Special:Contributions/CanonNi|contribs]]) 13:00, 24 May 2024 (UTC)
::But then you run into the irony that one of the greatest contributors, in terms of number of contributions, was recently banned indefinitely. [[User:AzseicsoK|Uporządnicki]] ([[User talk:AzseicsoK|talk]]) 13:29, 24 May 2024 (UTC)
:::You can still consider them a great Wikipedian. [[User:Gråbergs Gråa Sång|Gråbergs Gråa Sång]] ([[User talk:Gråbergs Gråa Sång|talk]]) 15:11, 24 May 2024 (UTC)
::::Numbers don't tell the whole story. [[User:Wolverine XI|<span style="color:#000080;">'''''Wolverine'''''</span> <span style="color:#8A307F;">'''''XI'''''</span>]] <sup>([[User talk:Wolverine XI|<span style="color:#2C5F2D;">talk to me</span>]])</sup> 15:38, 24 May 2024 (UTC)
:::::The essay [[WP:BEANS]] is probably relevant to the question of who was the worst vandal/troll. [[User:TSventon|TSventon]] ([[User talk:TSventon|talk]]) 15:43, 24 May 2024 (UTC)

== How to stop a person called CNMall 41 in simply changing and deleting list of programs broadcast by Asianet page for stupid reasons ==

Hi, I am Manisha Asianet who had been editing list of programs broadcast by Asianet since few weeks, I have been adding relevant sources, along with the programs that were been telecasted on Asianet as I have watched the shows since 2001-2002 period, but a user called CNMall 41 is unwantedly deleting my content as well as many other persons content on Asianet list that too the program that is been telecasted currently on the channel, cant understand whats wrong with him, he even claiming that I am getting money for publishing content as per Asianet's wish it seems by giving me a warning this matter for no reason. What to do [[User:Manisha asianet c|Manisha asianet c]] ([[User talk:Manisha asianet c|talk]]) 16:18, 24 May 2024 (UTC)
:@[[User:Manisha asianet c|Manisha asianet c]]: {{welcometea}} Rather than continue to [[WP:EDITWAR|edit war]] over the article in conjunction with an IP editor, you're better off making [[WP:EDITREQ|edit requests]] on the article's talk page, as you have confirmed a [[WP:COI|conflict of interest]] in [[Special:Diff/1221162254|this comment]] of yours, suggesting that you're doing it for promotional reasons ({{tq|[...] which I felt necessary to add to the list as Wikipedia has a wider reach}}). —[[User:Tenryuu|<span style="color:#556B2F">Tenryuu&nbsp;🐲</span>]]&nbsp;(&nbsp;[[User talk:Tenryuu|💬]]&nbsp;•&nbsp;[[Special:Contributions/Tenryuu|📝]]&nbsp;) 16:45, 24 May 2024 (UTC)

== My mother was a creator. I am a professional writer about creators. Can I create a page about her? ==

[http://charleenkinserdesigns.com/ Charleen Kinser Designs] is a company that my mother, Charleen Kinser, ran from 1977 to 2002. She passed away in 2008. I am a professional writer about creators. I would like to create a page about her and her creative work (I have the archives and am the only surviving family member) but have read that one should not create a page about a family member because of COI. In this case, I am the one most qualified to write about her. See [https://www.maggiehohle.com/ my website] for proof of my professional status in this industry. [[User:Maggiehohle|Maggiehohle]] ([[User talk:Maggiehohle|talk]]) 17:47, 24 May 2024 (UTC)

:@[[User:Maggiehohle|Maggiehohle]] It is not forbidden for editors with a [[WP:COI|COI]] to write draft articles for consideration by experienced reviewers. The process is described at [[WP:AfC]]. You should declare the COI and note that one of the things that Wikipedia is [[WP:NOTMEMORIAL|not, is a memorial site]]. You would benefit from reading [[WP:BACKWARDS|this guidance]] and note that we have many technical requirements for the way we do [[WP:CITE|citations]]. All this means that, despite your credentials, you may want to practice by editing existing articles before you take on the larger task of creating a wholly new one. [[User:Michael D. Turnbull|Mike Turnbull]] ([[User talk:Michael D. Turnbull|talk]]) 18:27, 24 May 2024 (UTC)
:@[[User:Maggiehohle|Maggiehohle]] Welcome to the Teahouse. My mother was also an artist and book illustrator, working mostly for [[The Bodley Head]] in the 1950s-70s. Like you, I am most qualified to write about my own mother, and I definitely think she was 'notable'. In my life, at least!
:However, nobody else unconnected with her has every written and published a detailed, in-depth account about her life and her work (including her work in a wartime office drawing undercarriages of [[Lancaster Bomber]]s). For that very simple reason, she fails Wikipedia's [[WP:NBIO|Noptability Criteria]] and so there can never be a page here about her.
:So, my question to you is to ask if other independent sources have ever published detailed, in-depth biographical articles about your mother and her work? If so, then maybe she might be seen as [[WP:GNG|notable]] by Wikipedia's criteria. If not, then she, too, would not merit an article here, irrespective of how wonderful her work may have been.
:We do try to discourage people from writing about people they know or are related too, as they tend not to be objectively written, and are rarely based solely on published sources. I might therefore try to encourage you to contribute your skills in writing and research to improving other areas of Wikipedia that you are not directly connected with. This does tend to lead to better-written, neutral articles. I hope this helps. Regards, [[User:Nick Moyes|Nick Moyes]] ([[User talk:Nick Moyes|talk]]) 18:28, 24 May 2024 (UTC)
::Hi, Nick. Thank you so much for your reply. This is helpful. Biographical articles have been written about Charleen Kinser in State College Magazine (1/91), Teddy Bear and Friends (4/98), with a shorter piece published in Contemporary Doll Magazine (4/93). Do these count? [[User:Maggiehohle|Maggiehohle]] ([[User talk:Maggiehohle|talk]]) 21:42, 24 May 2024 (UTC)
:@[[User:Maggiehohle|Maggiehohle]] Is your mother this Charleen Kinser? [https://archive.org/details/americanteddybea00mull/page/93/mode/2up?q=%22Charleen+Kinser%22] [[User:Gråbergs Gråa Sång|Gråbergs Gråa Sång]] ([[User talk:Gråbergs Gråa Sång|talk]]) 21:29, 24 May 2024 (UTC)
::Yes, that is my mother. [[User:Maggiehohle|Maggiehohle]] ([[User talk:Maggiehohle|talk]]) 21:47, 24 May 2024 (UTC)
:::Continued at [[User_talk:Maggiehohle#Charleen_Kinser]]. [[User:Gråbergs Gråa Sång|Gråbergs Gråa Sång]] ([[User talk:Gråbergs Gråa Sång|talk]]) 22:24, 24 May 2024 (UTC)

== General question about WP:Tea ==

So, I was looking at the “Other Areas of Wikipedia”, and the description for The Teahouse and the Help Desk seem very similar. Would it be a good idea for a noob/old rusty editor to assume that WP:Tea and WP:HD are roughly similar, with Tea being more reserved for basic questions? [[User:The Phase Master|The]] [[User talk: The Phase Master|Phase]] [[Special:Contributions/The Phase Master|Master]] 18:02, 24 May 2024 (UTC)

:[[File:Tea in action.jpg|thumb|Tea in action]]
:@[[User:The Phase Master|The Phase Master]] Welcome to the Teahouse! Yes - they're both very similar, but we do aim to welcome and support new users in particular, and in as friendly a way as possible. We have a less formal design layout here, too. And we do serve Tea to any editor, new or old. Here's one just for you! Of course, questions from anyone are always welcome. (The Help Desk does tend to take attract more technical questions, though.) Regards, [[User:Nick Moyes|Nick Moyes]] ([[User talk:Nick Moyes|talk]]) 18:14, 24 May 2024 (UTC)

== Fastest way to number a list article? ==

I need to number this list article - [[List of ethnic groups in Nigeria]] to be sure it is accurate, but I don't enjoy repetitive tasks so much. I wish there was an AI integration somewhere that can easily get this done for me...that's just a joke. My question is: what is the fastest way to number a list article such as this? [[User:HandsomeBoy|HandsomeBoy]] ([[User talk:HandsomeBoy|talk]]) 18:39, 24 May 2024 (UTC)
: {{re|HandsomeBoy}} It looks like you can use {{t|Row numbers}} for this. See that template's page for examples on how to use it. [[User:RudolfRed|RudolfRed]] ([[User talk:RudolfRed|talk]]) 18:47, 24 May 2024 (UTC)
: {{U|HandsomeBoy}} How will numbering help the reader or otherwise benefit the list? (And is a single page in a news website an adequate source for the use that's made of it here?) -- [[User:Hoary|Hoary]] ([[User talk:Hoary|talk]]) 21:18, 24 May 2024 (UTC)

== Same but different ==

I need a list that is the same as this one but that has no logos instead of including logos. I would like to work on adding company logos/emblems to any article for a company that is currently lacking them, but I don't know a good way to create that list and then sort through it. [[User:Iljhgtn|Iljhgtn]] ([[User talk:Iljhgtn|talk]]) 21:38, 24 May 2024 (UTC)

:"the same as this one": the same as ''which'' one? And are you looking for a list format (to which you'll add items), or for a ready-made list of items? -- [[User:Hoary|Hoary]] ([[User talk:Hoary|talk]]) 22:30, 24 May 2024 (UTC)
::Ready made list. Here is the link that I forgot to include with my question: [[:Category:Pages using infobox company with a logo from wikidata]] [[User:Iljhgtn|Iljhgtn]] ([[User talk:Iljhgtn|talk]]) 22:41, 24 May 2024 (UTC)
:::[[:Category:Pages using infobox company with a logo from wikidata]] [[User:Iljhgtn|Iljhgtn]] ([[User talk:Iljhgtn|talk]]) 22:41, 24 May 2024 (UTC)
::::{{ping|Iljhgtn}} We don't appear to have a specific category for '''company''' articles lacking logos, but many such articles should be in [[:Category:Wikipedia requested logos]], if you want to trawl through that extremely large category. [[User:Deor|Deor]] ([[User talk:Deor|talk]]) 23:39, 24 May 2024 (UTC)

== ARBECR restrictions on talk pages ==

I had a talk page comment outlining four simple edit suggestions for [[Talk:Casualties of the Israel–Hamas war|Casualties of the Israel–Hamas war]] removed with the removing editor citing WP:ARBECR. My understanding of the policy is that non-extended-confirmed editors cannot edit pages related to contentious subjects, but are allowed to make simple edit suggestions on Talk pages provided they are non-disruptive. Am I misunderstanding the policy, or do my edit suggestions go beyond what is allowed (and if so, why)? See the [https://en.m.wikipedia.org/w/index.php?title=Talk:Casualties_of_the_Israel%E2%80%93Hamas_war&diff=prev&oldid=1225495573&title=Talk%3ACasualties_of_the_Israel%E2%80%93Hamas_war&diffonly=1 removed request]. [[User:ExVivoExSitu|ExVivoExSitu]] ([[User talk:ExVivoExSitu|talk]]) 21:47, 24 May 2024 (UTC)

:Talk pages should be protected if non EC editors are to be excluded from participation, as is the case for [[Talk:Israel–Hamas war]]. Such protection does get rid of drive-by edit requests, but otherwise merely shifts consctructive suggestions to [[WP:RFPP]], which is an additional burden on administrators. I have reverted the removal of your comment, but it is likely the talk page may get protected. ~[[User:Anachronist|Anachronist]] <small>([[User talk:Anachronist|talk]])</small> 23:15, 24 May 2024 (UTC)
::Thank you for clarifying (and reverting), that makes a lot of sense. There is a lot of page traffic and some of it seems unproductive, so I think you're right. [[User:ExVivoExSitu|ExVivoExSitu]] ([[User talk:ExVivoExSitu|talk]]) 23:20, 24 May 2024 (UTC)
:::I have a different perspective. In my view, edit requests from non-extended confirmed editors should be entirely non-controversial, such as asking for typographical errors or obvious grammatical errors to be corrected. I see your requests as deep content discussions of a highly controversial topic. I recommend that you edit productively elsewhere until you meet the 30 day/500 edit standard. [[User:Cullen328|Cullen328]] ([[User talk:Cullen328|talk]]) 03:42, 25 May 2024 (UTC)

== Changing image in infobox for Tiangong Space Station article ==

Hello. I noticed that the article for the Tiangong space station currently has a simulated image as the main image in the infobox. Since lead images should be representative (https://en.wikipedia.org/w/index.php?title=MOS:LEADIMAGE&redirect=no) it seems that it would be better for an actual image to be there instead. There are complete images of the space station from the China Manned Space agency (https://en.cmse.gov.cn/dmt/tj/shenzhou16/) and the disclaimer for media on the CMSA website says that fair use is allowed. I was trying to upload one of the images to replace the current lead image but couldn't figure out how since I'm new to editing Wikipedia and was confused by only text appearing when trying to edit the infobox. I would greatly appreciate help. [[User:GoldenOrbWeaver|GoldenOrbWeaver]] ([[User talk:GoldenOrbWeaver|talk]]) 23:14, 24 May 2024 (UTC)
:Hello, {{u|GoldenOrbWeaver}}. Allowing for "fair use" is not enough. What is required is a robust, legally binding license that explicitly allows for unlimited use by anyone, anywhere, for any purpose at all including commercial uses, with the only restriction being proper attribution. The Creative Commons Attribution-Share Alike 4.0 International license is the most common license that qualifies, but there are others. [[User:Cullen328|Cullen328]] ([[User talk:Cullen328|talk]]) 01:41, 25 May 2024 (UTC)
::Wouldn't this fall under [[Wikipedia:Non-free content criteria|WP:NFCCP]]? All detailed images of the Tiangong (including the ones I previously linked) were released by the CMSA so there's no free equivalent, there's no market role being replaced, it seems like it's minimal usage if just one image is used, the photos have been published in various newspapers, it meets Wikipedia guidelines, it would be used in an article, and having an actual photo of the space station seems important to readers' understanding. I apologize if I'm misunderstanding this policy. There seem to be other images published by the CMSA in the article already also. [[User:GoldenOrbWeaver|GoldenOrbWeaver]] ([[User talk:GoldenOrbWeaver|talk]]) 05:28, 25 May 2024 (UTC)

== Best way to solicit feedback on an article? ==

Hello,

I am looking for feedback on one of my articles as it remains classified as a start class article and I am not sure what else is needed to get it reassessed to a C. I am looking for the best forum on wikipedia to solicit feedback.

The article is currently live under [[Big Excursion]]. I previously incorporated some of the feedback I received here about the title of the article and infobox. I am most concerned with ensuring I have no unsourced claims given the contentious nature of the topic, and while I believe I have done a lot to address that (40 references, many used in multiple places), I am sure there is much more to do when discussing such a topic. Luckily Bulgarians seem much more reasonable about this sort of thing than a lot of other peoples. [[User:Pietrus1|Pietrus1]] ([[User talk:Pietrus1|talk]]) 23:32, 24 May 2024 (UTC)

:If you are specifically looking to get feedback on an article, the first place I would look is in the talk pages of the WikiProjects relevant to that article, in this case [[WP:BULGARIA|WikiProject Bulgaria]]. For article assessment (and reassessment), you can ask at [[Wikipedia:WikiProject_Wikipedia/Assessment#Requesting_an_assessment]]. [[User:Reconrabbit|<span style="color:#6BAD2D">Recon</span>]][[User talk:Reconrabbit|<span style="color:#2F3833">rabbit</span>]] 23:37, 24 May 2024 (UTC)
::Thanks I asked for feedback and asked for and received a general assessment to know where it is at. [[User:Pietrus1|Pietrus1]] ([[User talk:Pietrus1|talk]]) 01:19, 25 May 2024 (UTC)

:[[User:Pietrus1|Pietrus1]], here's a paragraph:
::Owing tn the widespread use of the term "Big Excursion", in line with Wikipedia article title guidelines [[Wikipedia:Article_titles]], this article uses the term "Big Excursion" to refer to the ethnic cleansing of Bulgarian Muslims in 1989, but it is capitalized and presented in [[scare quotes|quotations]].
:Other than fleetingly, formulaically and inconspicuously ("see below", etc), Wikipedia articles don't describe themselves. Anyway, readers can see for themselves which term the article uses, and how this term is presented. So please remove the paragraph. (As for the question of "start" versus "C" ranking, the distinctions among "start", "C" and "B" were made decades ago, at which time they were useful; but since that time their utility has pretty much evaporated and these days few editors -- let alone readers -- much care.) -- [[User:Hoary|Hoary]] ([[User talk:Hoary|talk]]) 00:51, 25 May 2024 (UTC)
::I use the [[WP:RATER]] script to get an automatic rating of articles I write or improve, which I think is generally useful for stub/ start/ C. [[User:TSventon|TSventon]] ([[User talk:TSventon|talk]]) 01:10, 25 May 2024 (UTC)
::Thanks. I will remove that. [[User:Pietrus1|Pietrus1]] ([[User talk:Pietrus1|talk]]) 01:11, 25 May 2024 (UTC)
:::I will have to check it out. [[User:Pietrus1|Pietrus1]] ([[User talk:Pietrus1|talk]]) 01:12, 25 May 2024 (UTC)
::::I looked at [[Wikipedia talk:WikiProject Bulgaria]] and the last discussion I saw with a reply was in 2021, so your query there may well not be answered. WikiProject Wikipedia/Assessment is obviously more active [[User:TSventon|TSventon]] ([[User talk:TSventon|talk]]) 01:43, 25 May 2024 (UTC)

== can someone fix this ==

this article https://en.wikipedia.org/wiki/Meru_International_School,_Hyderabad looks really really messy and also its too promotional which isn't supposed to be allowed i think. maybe its written by chatgpt or smth. can someone fix it or maybe make it a draft idk how, so an admin can review it to make it nice and neat '''[[User:Freedun|<span style="color: #000000">Freedun</span>]]''' ([[User talk:Freedun|yippity yap]]) 05:27, 25 May 2024 (UTC)

:I just downloaded a code but it says "API error: cantmove" '''[[User:Freedun|<span style="color: #000000">Freedun</span>]]''' ([[User talk:Freedun|yippity yap]]) 05:30, 25 May 2024 (UTC)

:The first six sections cite no sources. I'm not an admin, but I have the power to move it to draft, and am tempted to do so. I wonder if that would be in accordance with policy? [[User:Maproom|Maproom]] ([[User talk:Maproom|talk]]) 05:37, 25 May 2024 (UTC)
::i noticed you made it look proper so thx for that '''[[User:Freedun|<span style="color: #000000">Freedun</span>]]''' ([[User talk:Freedun|yippity yap]]) 06:20, 25 May 2024 (UTC)

:It's now at [[Draft:Meru International School, Hyderabad]]. This is the second time that an article so titled has been draftified; but what's now the draft was created (or re-created) less than a week ago. -- [[User:Hoary|Hoary]] ([[User talk:Hoary|talk]]) 06:58, 25 May 2024 (UTC)

== What can I do? ==

? [[User:Calciocalcistico|Calciocalcistico]] ([[User talk:Calciocalcistico|talk]]) 11:30, 25 May 2024 (UTC)
:{{ping|Calciocalcistico}} Can you be a bit more specific? Do you have question about editing Wikipedia? Do you have a question about a particular Wikipedia article? Do you want to know how you can contribute to Wikipedia? Try taking a look at [[:Wikipedia:Contributing to Wikipedia]] and [[:Wikipedia:FAQ/Main]] to see whether the information contained on those pages is helpful. -- [[User:Marchjuly|Marchjuly]] ([[User talk:Marchjuly|talk]]) 11:53, 25 May 2024 (UTC)
::@[[User:Marchjuly|Marchjuly]] thank you so much [[User:Calciocalcistico|Calciocalcistico]] ([[User talk:Calciocalcistico|talk]]) 11:54, 25 May 2024 (UTC)
:Blocked for [[Wikipedia:Sockpuppet investigations/14 novembre|sockpuppetry]]. ~~[[User:2NumForIce|<span style="background:#007eb3;color:#ffffff;">2NumForIce</span>]] ([[User talk:2NumForIce|<sup style="color:#007eb3">speak</sup>]]<span style="color:55a7db">&#124;</span>[[Special:Contributions/2NumForIce|<sub>edits</sub>]]) 15:05, 25 May 2024 (UTC)

== Asking for feedback ==

Hello,
I am seeking feedback for my article that I want to publish on wikipedia. It got declined twice and I have put every effort to integrate the provided feedback by the fellow wikipedians. I want to re-submit the artice and I want to make sure this time it won't be declined. Kindly give it a read and provide me with feedback for improvement.

[[User:Akbarirazia/sandbox]] [[User:Akbarirazia|Akbarirazia]] ([[User talk:Akbarirazia|talk]]) 12:24, 25 May 2024 (UTC)

:As background, the page that was submitted was [[Draft:Amu Television]] so the versions that were rejected are in the history of that page. [[User:TSventon|TSventon]] ([[User talk:TSventon|talk]]) 12:46, 25 May 2024 (UTC)
:I don't think it's notable, and looks promotional in nature. [[User:Wolverine XI|<span style="color:#000080;">'''''Wolverine'''''</span> <span style="color:#8A307F;">'''''XI'''''</span>]] <sup>([[User talk:Wolverine XI|<span style="color:#2C5F2D;">talk to me</span>]])</sup> 14:16, 25 May 2024 (UTC)

== Question about date format ==

Hi, I am wondering why some of the dates are formatted as DD/MM/YY instead of MM/DD/YY [[User:GoodHue291|GoodHue291]] ([[User talk:GoodHue291|talk]]) 12:43, 25 May 2024 (UTC)

:@[[User:GoodHue291|GoodHue291]] Because a large part of the world uses {{xt|dd/mm/yy}} for dates; take a peek at [[List of date formats by country]]. For how Wikipedia deals with this, have a read of [[WP:DATEOVERVIEW]], with [[MOS:DATEFORMAT]] for guidance. [[User:Bazza_7|Bazza&nbsp;<span style="color:grey">7</span>]] ([[User_talk:Bazza_7|talk]]) 12:49, 25 May 2024 (UTC)
:{{Re|GoodHue291}} It depends on the country. There's no universal date format; the MDY format is mostly American, while the DMY format is much more present across the world. [[User:Wolverine XI|<span style="color:#000080;">'''''Wolverine'''''</span> <span style="color:#8A307F;">'''''XI'''''</span>]] <sup>([[User talk:Wolverine XI|<span style="color:#2C5F2D;">talk to me</span>]])</sup> 14:12, 25 May 2024 (UTC)
:Where do you see DD/MM/YY? It shouldn't normally be displayed in articles but can be used internally in some places. [[User:PrimeHunter|PrimeHunter]] ([[User talk:PrimeHunter|talk]]) 14:26, 25 May 2024 (UTC)
::{{smalldiv|1=I assume they mean they're seeing {{xt|day month year}} in prose rather than something like {{!xt|09/02/19}}. —[[User:Tenryuu|<span style="color:#556B2F">Tenryuu&nbsp;🐲</span>]]&nbsp;(&nbsp;[[User talk:Tenryuu|💬]]&nbsp;•&nbsp;[[Special:Contributions/Tenryuu|📝]]&nbsp;) 14:45, 25 May 2024 (UTC)}}
::There should be a line at the top of the page with something like {{tlx|Use mdy dates}} or {{tlx|Use dmy dates}}. You should use that date format to keep it consistent ~~[[User:2NumForIce|<span style="background:#007eb3;color:#ffffff;">2NumForIce</span>]] ([[User talk:2NumForIce|<sup style="color:#007eb3">speak</sup>]]<span style="color:55a7db">&#124;</span>[[Special:Contributions/2NumForIce|<sub>edits</sub>]]) 15:02, 25 May 2024 (UTC)

== Hi! I just joined Wikipedia! ==

It's great to finally be here!&nbsp;[[User:HowDoIPicAName|HowDoIPicAName]] ([[User talk:HowDoIPicAName|talk]]) 14:36, 25 May 2024 (UTC)

== Sourcing for "Novelizations" ==

Right now, the novelizations section of [[Beverly Hills, 90210]] is sourced to "thriftbooks", a retail website. This seems less than ideal.

What would the preferred type of reference for this be? Worldcat? Goodreads? The full text at the Internet Archive? A site like Fantastic Fiction? [[User:Walsh90210|Walsh90210]] ([[User talk:Walsh90210|talk]]) 15:14, 25 May 2024 (UTC)

:This might do:[https://archive.org/details/seriousaboutseri0000mako/page/8/mode/2up?q=%22Mel+Gilden%22+%22beverly+hills%22] [[User:Gråbergs Gråa Sång|Gråbergs Gråa Sång]] ([[User talk:Gråbergs Gråa Sång|talk]]) 16:51, 25 May 2024 (UTC)
:And btw, @[[User:Walsh90210|Walsh90210]], per [[WP:COI]] you shouldn't write about yourself ;-) [[User:Gråbergs Gråa Sång|Gråbergs Gråa Sång]] ([[User talk:Gråbergs Gråa Sång|talk]]) 16:52, 25 May 2024 (UTC)

== "I'm not a robot" check fail ==

Hello! I hope you're doing well.
Recently, I've encountered two issues while translating on my mobile phone:
1. I'm unable to publish my translations because the "I'm not a robot" test continually prompts me to type the displayed word, even after correctly entering it.
2. I'm unable to continue translations started on my mobile phone when using the desktop website on my PC. The "continue translation" button does not appear; only the "start translation" button is visible, which does not function correctly.
Could you please assist me with resolving these issues or recommend someone who could help? [[User:Vasconcelos-Giovanni|Vasconcelos-Giovanni]] ([[User talk:Vasconcelos-Giovanni|talk]]) 16:30, 25 May 2024 (UTC)

Revision as of 17:29, 25 May 2024

fffffffffffffff